Sunteți pe pagina 1din 89

DR.

STRANGE

MARCH RQS
Feb 28-March 1
CiCi
#Day_1 :a lot of perio, less ortho, only two q about space maintainer

1- Agenesis is least in
A. Third molars
B. 2nd premolars
C. Lateral incisor
D. Canines 👍

2. Common location of supernumerary teeth :


A. Max posterior
B. Man anterior
C. Man anterior
D. Max anterior 👍

3. Developmental anomaly of Supernumerary teeth occurs in which stage of tooth development :


A. Initiation 👍
B. Apposition
C. Bell stage

4. Apical extension of pulp chamber and blah blah is


A. Concrescence
B. Taurodontism 👍
C. Fusion
D. Germination

5. One of the following is manifested with seizure :


A. Hemangioma
B. Hereditary hemorrhagic telangiectasia
C. Encephalotrigeminal haemostasis 👍
D. One more option

6. Bilateral parotid enlargement in


A. Silothiasis
B. Pleomorphic adenoma
C. I can't recall it
D. Wharton tumor 👍

7. Pt complaints of inability to close the right eye after LA injection in rt side, the affected nerve is :
A. Ophthalmic
B. Facial 👍
C. Optic
D. Trigeminal

8. Silothiasis is common in
A. Stenson's duct
DR. STRANGE

B. Sublingual duct
C. Wharton's duct 👍
D. Minor salivary gland duct

9 and 10. Repeated twice but in dissimilar words, the procedure of apical closure in permanent immature non vital
tooth is :
A. Pulpectomy
B. Pulpotomy
C. Apexogenesis
D. Apexification 👍

11. Modeling for fearful child :


A. Show him the procedure in non fearful child (usually sibling) 👍
B C D (More 3 options)

12- dentist ignoring unacceptable behaviour is


A. Exposure
B. Extinction 👍
C D ( l can't remember)

13- dentist duties to practice truthfully, which type of ADA ethics :


A. Non maleficence
B. Justice
C. Autonomy
D. Veracity 👍

14. Informed consent is type of


A. Autonomy 👍
B. Justice
C. Beneficence

15- unethical to fee the patient solely bcoz he is benefiting from insurance blah blah blah
A. Veracity 👍
B. Justice and 2 more options

16- chronic periodontitis is more common in


A. Hispanic male
B. Hispanic female
C. Black female
D. Black male 👍
17- modified widman flap is
A. Partial thickness flap apical to mucogingival junction
B. Full thickness flap apical to mucogingival j
C. Partial thickness flap coronal to mgj
D. Full thickness flap coronal to mgj

18- systemic antibiotic and scaling are combined as main treatment for (not same statement but same meaning) :
A. Chronic periodontitis
B. Aggressive periodontitis 👍
C ANUG
D. Desquamative gingivitis
DR. STRANGE

19- DMF index for :


A. Caries 👍
B. Plaque
C. OHI - S
D- PI

20. Percentage of fluoridated community water in US :


A. 60 %
B. 74 👍
C. 80

21- the children between 7-10 years are mostly be beneficial in which of the following preventive method :
A. Diet counselling in dental clinic
B. Pits and fissures sealants
C. Removal of candy machine from school
D. OHI in class

22- band and loop space maintainer is all Except


Prevents vertical eruption of opposing tooth

Band and loop


-helps prevent the mesial movement of K (if L is missing)
-To prevent tooth crowding when the succedaneous counterpart erupts
- keeping remaining teeth in proper position will also guide the eruption of the permanent 1st molar.
-Space maintainers should not be placed in children below 3 yrs of age or younger.

23 - tooth and tissue support appliance :


A. Lingual arch
B. Nance 👍
C. Transpalatal arch
D. One more option

DD:
Tissue borne: Frankel
Tooth Borne: Activator, Bionator, Herbst appliance, twin block appliance

24 - pt has skin nodules, Supernumerary teeth and one more description, investigator should investigate for :
intestinal polyps
Peutz–Jeghers syndrome is an autosomal dominant genetic disorder characterized by the development of benign
hamartomatous polyps in the gastrointestinal tract and hyperpigmented macules on the lips and oral mucosa

25- x ray, there are unerupted teeth, retained


deciduous, impacted permanent teeth and l saw
osteoma also, Wt the diagnosis :
A. Ectodermal dysplasia
B. Odontogenic keratocyst
C. Gardner's syndrome 👍
D. One more option
DR. STRANGE

26. Treacher choline (mandibulofacial dysostosis) has a relation to zygoma in same manner of cleidocranial
dysplasia relation to :
A. Ribs
B. Clavicle 👍
C. Iliac

27. Macroglossia doesn't occur in


A. Amyloidosis
B. Down syndrome
C. Hypothyroidism
D. Hyperthyroidism 👍

28. Brown tumor occurs in


A. Hyperthyroidism
B. Hypothyroidism
C. Hyperparathyroidism 👍
D. Hypoparathyroidism

29- best time for ttt of pt under dialysis :


A. The day after dialysis 👍
B. Day before
C. Hours before

30- periodontitis in AIDS pt will be treated by all of following except


A. Scaling
B. Systemic antibiotics
C. Antifungal to prevent candidiasis
D. Multiple free gingival graft to correct the gingival defect or contour

31- dementia :
A. Short term memory loss
B. Long term memory loss

32- What is true about elderly pt :


A. Become less intelligent and difficult to change their behaviour or something like this
B. Senility comes by age

33 - evaluation of 2 groups A and B with 2 drugs for same period, what type of study
A. Cross sectional
B. clinical trials
C. cohort

34- one month ago, dentist study the prevalence of dental caries among school students, Wt study :
A. Cross sectional 👍
B. Cohort
And others

35- dentist studies 4 unrelated pts with myofascial pain and myalgia, he wants to publicate a papers, What type of
study :
A. Case series ( l chose this)
B. Clinical trial
C. Observational
DR. STRANGE

D. Experimental

36- mechanism of fetal alcohol syndrome


A. Failure of cells distribution
B. Neural crest apoptosis
C. Zygote implantation
D. I can't recall it

37- purpose of percussion :


A. To assess inflammation of pdl 👍
B. To assess pulp status

38- painful response that subsides quickly after removal of stimulus is


A. Normal pulp (1-2 sec pain then normal)
B. Reversible pulpitis (Exaggerated pain then after stimulus is gone it doesn’t linger anymore.)
C. Irreversible pulpitis
D. Pulp necrosis

39- polycrystalline ceramic :


A. Zirconia
B. Feldspathic porcelain
C. Aluminosilicate
D. leutice

Polycrystalline: ceramic contains no glass. Packed atoms making it tougher and susceptible to crack propagation.
Esthetic glass ceramic, while crystalline has higher strength

40- greatest fracture force


A. Zircon 👍
B. Lettice something
3- aluminium or lithium

41 - least fracture force :Feldspathic

42- 1-2 1/2 years children are non compliant with dentists bcoz :
A. Internal control is underdeveloped
B. Language is developing
C. Parental proven or something
D. Parental something

43- papoose is accepted except


A. 4 quadrants dentistry in cooperative Child 👍

44- osteoporosis is manifested as


A. Uncalcified osteoid
B. Unusual thin trabeculae 👍

45- desensitising agent acts by


A. Demineralizing the dentinal tubules
B. Remove the smear layer
C. Depolarizing the nerve 👍
DR. STRANGE

D. Killing the bacteria

46 - bisphosphonate uses in all of the following except :


A. Metastatic breast cancer
B. Metastatic prostate cancer
C. Hypercalcemia of multiple myeloma
D. Osteomyelitis 👍

Google: treat osteoporosis, osteitis deformans (Paget's disease of the bone), bone metastasis (with or without
hypercalcaemia), multiple myeloma, and other conditions involving fragile, breakable bone.

47- 2 years child, his mom said my son has sore gum and doesn't eat, his fever 101 F, Wt diagnosis
A. Normal eruption
B. Primary herpetic gingivostomatitis 👍

48- 15 years old pt with 107 f, lymphadenopathy, sore gums, diagnosis


A. Primary herpetic gingivostomatitis 👍
B. Infectious mononucleosis

49- pic of child with generalized swollen gingiva, bleeds easily, had history of recurrent skin infection, diagnosis
A. Herpetic lesion
B. Myelomonocytic leukemia 👍

50 - Case, pt said his cheeks is swollen, he came 1 month ago from Japan, had fever and fatigue followed by
swollen cheeks and muscle pain, dz
A. Dental abscess
B. Mumps 👍

51- distance between 2 implants. Answer: 3 mm


52 - distance between implant and adjacent tooth. Answer: 1.5 mm
53. Maximum local anesthesia 2% lidocaine with 100.000 epi in pediatric pt
A. 4.4 mg / kg
B. 7 mg/ kg

54- calculation of LA for pedo pt 16 kg. =16*4.4mg=70.4mg


55- most common cells in gcf: Neutrophils

56 - effect of smoking on periodontitis all Except


A. Increase gingival crevicular fluid
B. Increase vascularity in gingival sulcus
C. Suppress the neutrophils
D. Increase pathologic flora

57. Microorganisms in ANUG before initiation of necrosis


A. filamentous Rods
B. Spirochetes
C. Fusochetes
D. Cocci
DR. STRANGE

58- acid of zoe: Poly acrylic


statements true reg zinc polycarboxylate: Chemically bonded.

It’s under physical properties of GIC.

59- which of the following is correct about luting agent


A. Glass ionomer is less translucent than polyphosphate
B. Solubility of polycarboxylate is independent of water power ratio change
C. D ( two more option that l chose one of them)

60 - which is correct about the axial walls of gold onlay


A. Convergence of gingival wall toward pulpal wall
B. Divergence of gingival wall toward pulp wall
C. Sharp angle with pulp wall
D. Sharp with proximal walls

61- prevalence of cleft lip and palate in US is 1 to :


A. 600
B. 700
C.1000
D. 1200

Cleft Lip 1:1000


Cleft Palate 1:2000

62- 21 years old male, he extracted his lower left third molar, on the second day he feels feverish, with sublingual
swelling that elevate his tongue, affects the swallowing and breathing, diagnosis : Ludwig angina 👍

63 - stridor : Crowing sounds, Laryngeal obstruction.


A. laryngospasm 👍
B. Bronchospasm

64- cellulitis
A. Neutrophilia
B. Neutropenia
C. Lymphocytosis

65- immunoglobulin IgG :


A. Decreases phagocytosis
B. Antigens receptor 👍
C. Produces (or induces l can't remember) macrophages and neutrophils
66- recurrence is least in :
A. Ameloblastoma
B. OKC
C. Odontogenic myxoma
D. Adenomatoid odontogenic cyst 👍

67- x ray with radiolucent /radiopaque lesion in the left posterior molar ramus area , diagnosis :
A. Ameloblastic fibro odontoma 👍
B. Ameloblastic fibroma
C. Dentigerous cyst
DR. STRANGE

D. Okc

Ameloblastic fibro odontoma vs. Ameloblastic fibroma

68- x ray of Dentigerous cyst (impacted tooth with


radiolucency around the CEJ)

69- which of the following lesion is restrictly diagnosed from microscopic features
A. Ameloblastoma
B. Okc 👍
C. Fibroma

70 - pic of prescription :
name of pt
Date
Rx : Motrin 800 mg/ 2 TABLETS, every 8 HOURS, for dental pain, name of Dr......, Bill......
Q: what is the wrong of this prescription :

A. Frequency of dose
B. Number of tablets per dose (I chose this, it's one tablet per dose)
C. Bill something

71- diazepam uses in all Except :


DR. STRANGE

A. Emesis
B. Anxiety
C. Muscle spasm
D. Seizures

72- in molar area, the lingual flange of mandibular flange is determined by :


A. Genioglossus
B. Geniohyoid
C. Mylohyoid 👍
D. Buccinator

73- the mandibular flange ( lateral to retromolar area) is determined by :


A. Buccinator
B. Masseter muscle 👍 notch

74- occlusal morphology determinants, what is correct :


A. Steeper articular eminence, flatter posterior cusps
B. Deeper overbite, flatter posterior cusps
C. Increased overjet, flatter posterior cusps 👍

March 1, 2019

1.Implant to tooth distance- 1.5mm tooth to tooth 3mm


2. Nitrous limit for kids- kids. 50% Nitro/50% O2.
3.Caries prep depends on what- Extent of decay
4.BZD antagonist- flumazenil
6. Cementoblastoma x-ray- radiopaque with a radiolucent halo, root is not distinguished

7. Electric shock- decrease VDO


8. CAOH is used for- remineralization, canal medicament, root dressing
Stimulates Secondary odontoblasts to repair with dentinal bridge formation.

9. How syneresis happens? -expulsion of water molecules from dental material leading to shrinkage after imp
is kept out for longer time. Eg. alginate

10.Port Wagner syndrome- Wagner syndrome is a hereditary disorder that causes progressive vision loss.

11. Pierre Robin syndrome- a hereditary disorder, micrognathia, glossoptosis, high-arched or cleft palate.
DR. STRANGE

12. Indirect retention- used in kennedy 1,2 distal extension prevent dislodgement of framework rpd.

13. Black male - chronic perio


14. Initial caries bacteria- s.mutans
15. Later on caries bacteria- lactobacillus
16. Kid with poor oral hygiene, what kind of ortho given- Removable
17. What force to do when making CD impression on masseter- forceful clench or functional tongue movement
that's lingual border
18. Non arcon vs arcon- arcon fpd, non arcon rpd cd
19. X Ray too white image. Why?- not exposed-low temperature-old developer-low kvp
20. Tb every year ? t/f- true
21. Tongue left side blue what is that?-Lymphangioma one side. Hemangioma
22. Child loss primary canine prematurely why?- premature loss of mandíbular primary canines reflect insufficient
arch size in anterior region. As such the crowns of laterals incisor during eruption impinge on primary canine roots
and cause them to resorb. When the canine is shed the midline will shift in the direction of the lost tooth. And also
lateral and lingual migration of mandibular incisors.

23. Which fluoride in porcelain?- NaF


24. Why is Low fusing porcelain used-
high fusing porcelain teeth
medium fusing-jacket crown
metal ceramic low fusing-pfm crown

25. Amalgam mai .5 mm marginal defect what tt approach- marginal ridge premolar 1.6mm molars 2mm , redo seems

26. Non vital canine diffuse swelling what tt- i think incision drainage first if fluctuant swelling, antibiotics, rct

27. paget's x ray- cotton wool


28. nutrient canal x ray- -blood vessels and nerves that supply the teeth, interdental spaces and gingiva.

29. okc or myxoma x ray doubt- myxoma multilocular-soap bubble. Okc-unicolar?!


Myxoma: poor, well demarcated associated with unerupted teeth, high rate of occurrence and
treatment: curettage

OKC: well radiolucency with smooth margins and thin radiopaque , high recurrence rate, T:
excision
DR. STRANGE

30. Cri-du-chat -Cri du chat syndrome, also known as chromosome 5p deletion syndrome, 5p− syndrome or
Lejeune's syndrome, is a rare genetic disorder due to chromosome deletion on chromosome 5

March 2

1. Age for hpv vaccine- before 18


2. Hpv associated scc location-oropharyngeal
3. Preg female urinate freq, why- pressure on the bladder

4. Many questions on rpd, rests -rest for vertical support can be on occlusal ,,inscsisal ,,lingual of canine

5. Radiographic landmarks- submandibular fossa, common meatus, hyoid bone, scale in late cephalopod, palate on
lat cephalopod =ruler is used for magnification In ceph

6. specks on film why?, light x ray why?-


Specks: sensitivity area on film
Light x ray due to overexposure

7. Lesions- odontogenic myxoma, Paget, ossifying fibroma

8. white lesion on floor of mouth. Answer: ORAL LEUKOPLAKIA

9. File diameter calculation- file size mm + File taper (D16mm)

Example:
K file- .02 taper

D1= diameter
Size 15 file-> 0.15

D2= diameter 16mm


Size 15K file -> 0.15mm + .02(16mm)= 0.47

10. La dose calculation

Maximum total dose:


lidocaine(xylocaine) & mepivacaine(carbocaine) =300mg
7 mg/kg adult
4.4 mg/kg child

Maximum total dose:


Articaine (Septocaine)= 500mg
7 mg/kg adult
3.2 mg/kg child

Maximum total dose:


Prilocaine(citanest) = 400mg
6 mg/kg adult
6 mg/kg child

Maximum total dose:


DR. STRANGE

Marcaine (bupivacaine)= 90mg


2 mg/kg adult
1.3 mg/kg child

Max carpules:
8.3 carp = Lidocaine (xylocaine) 2%
5.6 carp= Mepivacaine (carbocaine) 3%
6.9 carp= Septocaine (articaine) 4 %
5.6 carp= Prilocaine (citanest) 4%
10 carp= Marcaine (bupivacaine) 0.5%

11. Who controls dental office material- ADA


12. Sample size irrelevant in which study- cross sectional
13. Epithelium growth per day- 0.5-1mm per day.
14. Low fusing porcelain used why? For crown and bridge

15. Porcelain occlusal adjustment, which bur? -diamond


16. Margin discoloration of veneer- Silver whole crown, Margin-Copper
17. Green discoloration of porcelain crown- silver does glazing . Copper green.

18. Anb-
Normal is 2
More than 4-cl2
Minus 0-cl3

19. Battery- doing a procedure without patients consent

20. Mm house pt classification, who is best pt


- best patient is philosophical
Other types hysterical
It’s for CD patients

21. What's the disadvantage of single implant- rotation


The screw is overcharged cause it is not ferulize with other teeth.

22. What's inside implant-


external internal both have hex
External is more common used.
External has risk of fracture something abutment
Internal causes fracture
At crest
DR. STRANGE

23. Opium side effects except (diuresis was in option)

24. Both diazepam & narcotics antagonist- flumazenil and naloxone

25. Cocaine addict pt, what u worry abt- infection


26. Hue value chroma
hue most difficult to change
Hue is 100
Hue is wavelength

Chroma saturation
Chroma least important
We can increase it

Value is lightness or darkness


cannot increase but we can decrease by adding complementary color

27. bluish discoloration near lingual feral attachment- ranula


28. Blush lesion on lateral border of tongue: Hemangioma
29. Melanin pigmentation: Menalonit
30. Traumatic neuroma: Answer: cause pain. Mental nerve or incisive papilla
31. Cantilever- one side is with no abutment
32. Short crown- pfm
33. Amalgam .5mm marginal defect, no caries, what to do- Observe
34. Hiv patient, necrosed canine, swelling orbital, what is first step: Antibiotic
35. Tegdma- Increase fluency cause BISGMA is too dense
36. Gracey curette-
offset Blade 60-70 working angulation
11 12-Mesial
13-14- Distal

37. If we grow undifferentiated mesenchymal cells in lab, what is most difficult


A. bone
B. cartilage
C. muscle
D. epithelium

38. Synchondrosis - Cartilage-like epiphyseal plate


39. Gardner- Multiple osteoma-polyps, supernumerary teeth
40. Chronic perio-black male
41. Elevators of tongue- styloglossus, palatoglossus
DR. STRANGE

42. Primary support for maxillary denture . Answer: Palate


43. Trismus after extraction, which muscle- medial pterygoid
44. IAN failure why- infection/low PH
45. Gorlin- Calcified falx cerebri-multiple oks-nevoid basal cell carcinoma
46. Why midline shift- Canine loss, missing teeth
47. Why anterior crossbite- Answer: Prolonged retention of primary teeth

( DD #11)
Anterior crossbite SHOULD BE CORRECTED before it reached the occlusal plane (while it was erupting)
Etiology factor: Prolonged retention of the primary teeth

Remember: Anterior crossbite in a Primary Dentition usually indicates a SKELETAL GROWTH PROBLEM.
NOTE: The Permanent anterior tooth that is MOST OFTEN ATYPICAL in size is the Maxillary lateral incisor.

48. Rubber dam doesn’t protect from what- mercury vapor inhalation
49. Surgery for open bite- lefort 1
50. Headgear that extrudes teeth- Answer: cervical headgear extrudes the molar.

high pull headgear: intrudes the molar … control maxillary molar eruption.

51. Single tooth recession, why: Plaque and occlusal trauma in one teeth.
52. Most radioresistant- Muscle-nerve
53. What fluoride to pt with porcelain- 2% neutral fluoride
54. X Ray exposure of one opg is equal to: 4 bitewings compared on amount
The amount of radiation on a panoramic RX is compared to?
1. A CT scan
2. MRI
3. 4 bitewings

4. Full mouth series of rx

55. Y shape over max lateral & canine: Line of Ennis

Rocky Fireball- March 5, 2019


DR. STRANGE

1. Study to know the effect of gastric bypass surgery on nutritional status:

- Observational
- Clinical trial
- Descriptive
- Cohort

Cohort study—study where there is more than one sample/cohort, and evaluations are done to
see how certain risk factors the groups have are related to developing a certain disease.
Cohort (prospective) studies - look forward from exposure to disease development

2. Most common tooth to get plaque


A mandibular incisors
B maxillary molars
C mandibular molars

3. Prevalence in which study.

For a population, the research divides the number of disease cases by the number of
people. By so doing, this investigator will have calculated which of the following rates.

4.When fail and safe mechanism start in nitrous at what percentage . Doesn’t mention if for kids or adults

Answer: 70%

5. How much oxygen give when u patient feel nausea in nitrous oxide.
A stop nitrous
B give 60% oxygen
C give 100 % oxygen

6. Mitral valve prolapse patient. Do you do premedication


Answer: No

7. You can check all drugs in complete blood count except


A. Clopidogrel
DR. STRANGE

B. Plavix
C. Apixaban
D. Warfarin

8. When u put crown on for cementation. What u check


Proximal contacts, occlusion there is one more which I never heard about was in every
option . Its something like iglu long word. Don’t remember

Answer: internal, proximal contact


9. Patient on initial stage in leaving smoking all can be done except
A Chantix
B hypnosis
C Nicotinic patches
D Bupropion

10. Patient came to you and he only speak limited English . You do all except
A Tell him pros and cons.
B Get consent in his own language
C You talk to spouse about his plan

11. A person come to you after SRP. He don’t have any calculus but his pocket depth is around 5 or 6 mm
generalised pocket . What u do
A Put him on maintenance phase
B. Do another SRP
C. Do surgery?

12. Lower 1st molar have through & through furcation involvement which was grade 3. What u do
A. Hemisection it and make it like premolar
B. Bridge from premolar to molar
C. Extract and implant

13. For external crevicular incision where it will end


A apical to alveolar crest
B Sulcular epithelium
C. Bone

14. Ectodermal dysplasia seen most in


A maxilla
B mandible
C Alveolar bone

X rays
1. stafne but they mention lingualized salivary gland.
2. Ear lobe.
3. Hyoid bone
4. Nasal process of maxilla.

Day 2 was mixed . A lot of questions on implant, prosthodontics,


Pano of patient showing if she have chronic sinusitis or not . Its like true false statement.
Oral pathology was straight from tuft pathology
They mention histopathology of one of the cyst but that’s mention in tuft pathology
DR. STRANGE

Charlotte

1. Age for hpv vaccine


Answer: 11-12 before 17

2. Hpv associated scc location. Answer: Soft tissue of oral cavity or laryngeal cavity

3.Preg female urinate freq, why. Answer: pressure on bladder

4. white lesion on floor of mouth, what can’t be dd


5. File diameter calculation. Answer:

6. La dose calculation
7. Who controls dental office material. Answer: ADA
8. Sample size irrelevant in which study. Answer: series/History
case
9. Epithelium growth per day- Answer: 0.5-1mm
10. Low fusing porcelain used why? Answer: True

High fusing= denture .


Medium fusing =all ceramic and jacket crown .
Low fusing =metal ceramic crown

11. Porcelain occlusal adjustment, which bur? Answer: Diamond

12. Margin discoloration of veneer.


Answer:
day-amine
Week-microleakage
Month-microcrack

13. Green discoloration of porcelain crown

14. Battery👍- Answer: no consent form


15. Mm house pt classification, who is best pt. Answer: philosophical

16. What's the disadvantage of single implant


17. What's inside implant. Answer: Hex

18. Opium side effects except (diuresis was in option)

19. Both diazepam n narcotics antagonist


Answer:
Diazepam - flumazenil
Narcotics- naloxone.

20. ANB
2 (+-2)
DR. STRANGE

> class II
<class III

21. Cocaine addict pt, what you worry about. Answer: infection
22. bluish discoloration near lingual feral attachment: answer: Ranula

23. Bluish lesion on lateral border of tongue- Hemangioma


24. Melanin pigmentation: Melanolit
25. Traumatic neuroma: Answer: cause pain. Mental nerve or incisive papilla
26. Amalgam .5mm marginal defect, no caries, what to do
Answer: observe
27. Cantilever: No distal abutment
28. Hiv patient, necrosed canine, swelling orbital, what is first step: Antibiotics
29. TeGMA. Answer: reduce viscosity created by bis gma

30. Gracey curette.


Answer:
1 cutting edge
Off set: 60-70 •
Working Angulation: 45/90

31. If we grow undifferentiated mesenchymal cells in lab, what is most difficult.


A. bone
B. cartilage
C. muscle
D. epithelium

32. Gardener.
Answer:
-We should worry about polyps
-Multiple odontomas
Impacted and -supernumerary teeth
Cotton wool appearance
Desmoid tumors (fibromatous)

DD

Gardner syndrome is an autosomal dominant disorder and is characterized by intestinal polyposis, multiple
osteomas, fibromas of the skin, epidermal and trichilemmal cysts, im-
pacted permanent and supernumerary teeth, and odontomas. The most serious complication of Gardner's syndrome
is the multiple polyps that affect the large intestine. The inevitable outcome of this disease is invasive colorectal
cancer.

33. A.k.a Gorlin syndrome.


Answer: keratocystic odontogenic tumor
calcified falx cerebri

KOT. It is also known as the basal cell nevus-bifid rib syndrome,


the basal cell nevus syndrome, and the Gorlin-Goltz syndrome.

34. Elevators of tongue. Answer: Styloglossus and palatoglossus


35. Primary support for maxillary denture (ridge not in option) Answer: lost teeth
DR. STRANGE

36. Trismus after extraction, which muscle. Answer: Medial pterygoid


37. Ian failure why. Answer: muscle and nervous tissue

38. Why midline shift: Missing teeth


39. Why anterior crossbite: Answer: Prolonged retention of primary teeth
40. Rubber dam doesn’t protect from what. Inhalation of Mercury
41. Surgery for open bite: Le Fort 1
42. Surgery for vertical maxillary excess:
43. Headgear that extrudes teeth: Cervical Headgear
44. What concern with LA to meth addict : Infection
45. Single tooth recession, why: Plaque & Occlusal trauma
46. Most radioresistant: Nerve-Muscle

March 5, 2019 Nbde Target

1. Patient chief complaint I can't chew. She had maxillary hemisectomy what will u do
a. Fabricate rpd
b. Refer prostho os for obturator
c. Refer os
d. Refer pros same refer n obturator for speech given

2. 3 months ago study done cancer among population


a. retrospective
b. observational
c. case series
d. cohort

3. for how long hcv stays on surface


a. 12 hours
b. 3 hours
c. 6 weeks
d. 3 weeks

4. congenital anomaly occurs or incidence of congenital disease increased by


A. mitogen
B. teratogen
C. cancerogen
D. immunogen

5. while choosing shade for anterior crown except


a. time
b. strength of ceramic
c. color related 2 options

6. pulmonary emphysema 4 options split like


A. constriction of air spaces
B. dilation
C. constriction and inflammation
D. again distal space constriction

7. cracking sound –emphysema, pneumonia, bronchitis, asthma


DR. STRANGE

8. patient with xerostomia comes for normal check up 4 caries next when routinely
a 3 months
b 6 months
c 12 months
d 18 months

9. worrisome for dentist


a. bulimia
b. anorexia
c. 2more
10. 9year old same rq
a primary herpetic
b anug
c stomatitis

11. pictures
I. xray of stylohyoid eagle,they put arrow and mention patient has pain in head and neck while opening mouth a
lesion was on palate for 6 weeks. And now on tongue red, given like median lesion. I picked candidiasis other
options tb, syphilis, histo didn’t make sense

II. acute leukemia picture, they describe like bruises on knee, bleeding u will recognize

III. erosive leukoplakia came read description , sloughing off membrane reddish same like dd picture

IV. 3-4 nodule like lesion came together fixed on tongue let me find pic it just said 4 options benign tumor, malignant ,
normal, one thing more ( I put benign it didn’t seem normal at all and malignant scc also not as no ulceration , lateral
border tongue}

V. picture of swollen lips, description like fever ulcer malaise (i'll find picture it was weird)

VI. picture of pterygomaxillary fissure PTM tear drop shape very faint focus on arrow

VII. picture on varices tongue it's dark black given n looks like nevus etc don’t pick go for varices 55yr old aged.

VIII. Multiple myeloma , punched out skull, calvaria changes given.

IX. 1-2 herpes related pictures.


I am finding i'll send if I see same.

12. tb test- yearly

13. dentist leaves patient=abandonment two times came

14. lingual inclines on buccal cusp of mandibular which movement

a centric
b protrusive
c working
d non working

15. minimal crown root ratio


A. 1:2 ideal
DR. STRANGE

B. 2:3 realistic
C. 2:1
D. 1:1

16. one graph came low,high,medium income , value of teeth for job interview
x=incidence y axis= income inequality something
a. high income
b. high income inequality
c. low income
d. low income in equality

it was a bar graph check a button on right side below review/exhibit to open it ok.

17. pharma= aspirin irreversible inhibit platelet, inhibit COX

18.pregnant woman 5 months , cavity which LA?


a. articaine
b mepivacaine
c lidocaine
d prilocaine

18. tetracycline stops penicillin how 4 big options saying


A. renal clearance inhibits
B. increase metabolism
C. increase absorptions
D. last said tetra inhibit growth synthesis

Pen-Bactericidal, tetra- Bacteriostatic

19. penicillin work on peptidase in cell wall

20. Parent says my child 7yr old bleeding gums while brush
a chronic gingivitis
b pubertal
c anug
d leukemia

21. they put mini case history like medical history one line, chief complaint one line and a question related just 20
questions came

like avulsed tooth 3 days back fell off bicycle, it was in milk and reimplanted in within one hour. Endo

Radiolograp shows splint well tooth 9, what will u do?

A observe
B remove splint
C endo
D related to endo something

22.pt says my child went Africa got bleeding gums since a months boggy gums
a. scurvy
b. rickets
DR. STRANGE

only scurvy make sense

23. pt says my child has adhd take ampheta what won't happen
a. saliva less
b. tachycardia
c. epi toxicity
24. pt says, my son fell when he as 8 and his tooth 8 avulsed it was replaced with implant at age 15 with crown
implant but I don’t like his smile examination tooth 8 is shorter than tooth 9
a. hyper eruption of tooth 9
b. extrusion of tooth 8
c. extrusion of 9
d. facial growth

25. ortho appointments in adult patients should be considered on


a. business schedule
b growth
c 2 more if i'll remember I'll write

26.non rigid minor connector why


a. parallel path not possible
b forces balance
c,d forget sorry

27. before placing fpd, tooth max 1st molar supra erupted you restore it why
a. micp
b occlusal contact
c forces balance

28 same asking in different way when you put fpd why you check contact
a so that you don’t have to grind later
b,c,d related to 27

29. You do 3 surface composite insurance company pays for one: BUNDLING

30. You increase the bill for full mouth x ray for
a. reimbursement
b upcoding
c bundling
d downcoding

31. veneer better than porcelain for Metal ceramic then answer is A
a. tooth conservation

32. A lot of endo but tiny question basics , related to pain on cold, thermal pain, pulp necrosis=10 questions, they turn
twist but nothing difficult read each word highlight and stay focused,

33 mechanism of action of propranolol and statin:


Propranolol- B blocker
Statin: HGM-COa lipid lowering medication
DR. STRANGE

34.aggressive and chronic periodontitis difference. Aggressive has vertical bone loss no plaque and neutrophils
dysfunction.

35. function of rest- when rest is used as an indirect retainer it is placed as far away from the edentulous area. TRUE

36. Pterygomandibular Raphe is made up of- buccinator and superior constrictor muscle ( 2 questions). TRUE

37. read about a, b points that are located on ortho ceph

38. In Inferior alveolar nerve block, the needle is injected into buccinator..learn where the needle should be injected
(anatomical relations)

39 features of cleidocranial dysplasia- supernumerary teeth


40 features of pierre robin syndrome- Cleft palate, retrognathia, macroglossia, glossoptosis, micrognathia
DR. STRANGE

41. thinning of hair is seen in- hypothyroidism


42. Atypical pseudocholinesterase related to articaine 4 times
It’s an amide but metabolized by Serum esterases, These should be cautiously used or not at all with
compromised Liver function
43. BZD antagonist = Flumazenil
they don’t ask directly they give a scenario and give bzp or opioid then ask what to give like midazolam overdose
type.

44. Opioid antagonist =same here naloxone not directly


45. Rubber dam question = teeth dry one
46. Which caries have seen increased trend=root caries

47. Child loses primary canines prematurely then what happens


A. rotation of teeth
B. ectopic eruption
C. delayed eruption

48. Which fluoride in porcelain for demineralization of tooth?rest of the teeth neutral Naf or fluoride varnish for
abutment

49. Bisphosphonate wala pt came with unrestorable tooth


Ans:
A. endo
B. coronectomy one

50. Tooth supported rpd rest


A. Away from ridge
B. Near ridge
C. On molar
D. On canine

51. Herpes cause


● Measles
● Chicken pox – ans
● Cat scratch
● Mumps
● Herpangina

52. Angina classification and ASA classification big question saying patient used nitroglycerin before 1 year now uses
patches 2-3 daily so pick unstable

● stable - at exertion
● unstable - at rest
ASA IV

53. Exception question about Cervical pull gear actions it said which movement have reciprocal on teeth.
Banding, z spring etc

54.How many milligram of fluoride in 1 ppm was there – ANSWER: 1 mg / L

55. Space maintainer for a patient 8 years old with both primary 2nd molars missing ?
● Band and loop
DR. STRANGE

● Lingual arch
● Distal shoe
● No space treatment needed

56. In skeletal class III patient what cephalometric value decreases as he gets older
● Sna
● Snb
● Anb

57. A 40 years old patient bilateral posterior crossbite , what would be the way to correct the problem
● Hyrax
● Maxillary Osteotomy
● Quad helix
● Ans is quad helix confirmed from ortho. We don’t know max osteo for dental crossbite in adults it's safe to
choose helix, checked pubmed also..

58. Patient had a root canal 2 years ago and now you see an apical lucency. What to do pt asymptomatic
a. Six months recall to take x ray and observe
b. Retreatment only if it becomes symptomatic
c.d not important

59. patient had trauma 2 year back now you see radio in anterior-mand. What to do same but different options

a. stop treatment n monitor

b. extract mand anterior and fpd

c check pulp vitality and do rct for nonvital

d. do all rct

60. Had that gic question it does fluoride and what else pick this “ionic bond to dentin and enamel

61. which property porcelain has?


A shear
B compressive
C tensile

62 reason for changing an amalgam


● Ditch of margins
● Corrosion
● Recurrent caries- ans

63. Amalgam sealing quality question. You did amalgam but if at times leakage how to define in reality
● Ans-Improves over time

64. The characteristic of an impression material to change its viscosity when applied tension ?
Thixotropic- true (thin to thick under sheer pressure)
DR. STRANGE

65. Dentist is doing a right mandibular block and after the patient states that he feels the right side towards the tip
anesthetized but not his lower lip .what should the dentist do?
● Deliver a long buccal
● Deliver a mental foramen
● Give pulpal anesthesia
● Give again ian block
● Answer in files is mylohyoid that is not given in options

66. a draining from tooth =Parulis

67 Flabby tissues in anterior maxilla for a complete denture impression “


A. do mucositis
B. passive technique

perio=lateral flap, fgg, chronic aggressive difference,


endo=diagnosis making

68. dentist says to bill u don’t have to be scared etc

a. Empathy
b. Irrelevant
c. Judgmental
d. 2-3 more words

69. what is most important in oral communication

a. how u spoke
b. when u spoke
c. what u spoke
d. where u spoke

70. adhesion is bad in sclerotic dentin: True

71. 5-7 ques on root caries related to shiny glossy dentin or soft caries or arrested caries on abrasion they kept
asking in many ways. SHINY HARD GLOSSY DARK: ARRESTED

72. anterior crossbite causes. Prolonged retention of primary Maxillary Incisor

73. rubella encephalopathy is acquired: CONGENITAL

74. 10-15 will be on gardener,ecto,pier,cleido.


Gardner- polyps of intestine, multiple supernumerary teeth
Ectodermal dysplasia-child appear to be more older, sparse hair, no sweat gland
Cleidocranial- aplasia of clavicle, cranial bossing, retained remaining teeth
Pierre robin- Micrognathia, Glosoptosis, cleft palate

75. hypercementosis seen wherea hyperparathyroid, paget


b giant cell granuloma
c all related things

76. papillon lefevre one ques: premature loss of primary teeth

Nbde Target -Day 2

1. Andreas - I got 12 cases


DR. STRANGE

2. Epilepsy picture click phenytoin and systemic disease


3. Prostate action on bladder which receptor: ADRENERGIC
4.amalgam tattoo pathetic picture just faint pencil type grey mark 👍
5. Cephalometric palatine process boundary between roots
Ruler in ceph
6. Nose pin in iopa
7. Drug drug basic interaction with propranolol epi types
8. Watch canine class 3,2, relations

9. Advices = write basic drug smoking allergy on notes 👍 as u have to open 4 separate windows a-history dental
history photographs the cephalometric or radiographers or pano all window don’t open same time and u need to see
picture to answer so write little rough hand basics so u have case in mind before attempting
10. Diabetes type 1 in kid
11. Asthma 2-3 times
12. Ortho weird cases
13 Prostho case
14. Perio furcation type
5 easy 6 hard 1 andreas
15. Iopa put eyes inside screen as iopa is small
16. Photos pano are good don’t fear
17. Lichen planus
18. Granuloma/ fibroma/ pleomorphic type in identity
19. Studies question Cohort
20. Rpd fpd post core crown related questions is post core ok
21. Overhang in amalgam
22. Calculus in radio
23. Counting Permanent teeth in pedo graph
24. Be speedy as opening closing windows thinking may take time 2nd set was hard
25. Patient from Mexico can speak Spanish immigrant
26. Watch few X-ray faults pano identify on google
27. (Unicorn + Kaplan )must few Asda cases if time
28. Watch lot of google pics text me I can send .
DR. STRANGE

Rocky Fireball March 5


1. 22 Year old patient with isolated discrete ulcers in mouth and palate. Also has elevated temperature, malaise
A. primary herpetic
B. aphthous

2. Ledge is least likely to be formed in


A. small canal
B. long canal
C. curved canal
D. short canal

3. when you try to seek more suspected information


A. structured question
B. probing

4. dentinal nuclei displacement in odontoblast


A. thermal irritation
B. mechanical irritation
C. desiccation

5. side effect of nitroglycerin: Throbbing Headache


6. spectral wavelength

-hue
-value
-chroma

7. most critical for etiology of caries


A. bacteria
B. saliva flow
C. ph of saliva
D. sugar consumption

8. Asthma:

9. Anaphylaxis:

X ray -fibrous dysplasia, ear lobe, incisive canal cyst

10. most diagnostic test for lichen planus


A. Ana antibodies
B. Fluorescence
C. Biopsy

11. true about dual cure cement


A. cement of choice for light veneer
B. reactivation of light is necessary for initial cure se

For INLAYS AND CERAMICS


dual cured resin cements need an adequate amount of light to initiate the polymerization process.
DR. STRANGE

12. If a particular test is to correctly identify 95 out of 100 existing disease cases, then that test would have a
A. Specificity of 95%
B. Sensitivity of 95%
C. Positive predictive value of 95%
D. Validity of 95%

13 Disadvantages of oral sedation include which of the following?

A. Multiple drugs needed for proper sedation


B. High incidence and severity of adverse reactions
C. Unpredictable absorption of drugs from the GI tract
D. Short duration of action

14. most congenitally missing tooth:


3 molar, 2 premolar, Maxillary Lateral
Most carious : 1 lower molar

15. most carious: Mandibular Molar

16. most periodontally involved: Maxillary 1st molar

17. dens in dente in which tooth: Maxillary Lateral

18. dentist not behaving well with colleagues and staff


A. beneficence
B. non maleficence
C. veracity
D. justice

19. taking out amalgam . which ethics. Answer: Veracity

20. returning patient back to specialist. Which ethics: Answer: JUSTICE

Non maleficence: ✓The specialists or consulting dentists upon completion of their care shall return the patient, unless
the patient expressly reveals a different preference, to the referring dentist, or, if none, to the dentist of record for
future care.
JUSTICE- ✓If treatment is provided, the dentist, upon completion of emergency treatment, is obliged to return the
patient to his or her regular dentist unless the patient expressly reveals a different preference.

21. 2 year old not behaving


A. papoose board
B. immobilize with help of assistant
C. immobilize with help of parent

22. best to treat 8,mm overbite: Lefort 1

23. posterior cross bite in 40 year old


A. Maxillary osteotomy
B. Quad helix

23. base of incision in gingivectomy


DR. STRANGE

A. Apical to base of pocket


B. Just coronal to mucogingival junction

24- medicine in acuter status epilepticus


A. Phenytoin
B. Midazolam

25. amantadine is. Answer: Antiviral


26. what is true about penicillin

A. Broad spectrum
B. Low toxicity

27. epinephrine interaction. With Thyroid: Increased heart rate

28- hyperpyrexia,. Tachycardia, nausea vomiting diarrhea, failure of cardiac output

A. Lesion in hypothalamus
B. Thyroid storm

29- side effect of persistent opioid drug abuse

A. Chronic cough
B. Diuresis ?
C. Respiratory stimulation
D. Xerostomia?
30. most critical in color selection
Value
- hue
Chroma
31. intensity, saturation of color: Chroma
32. Child is taking .7 ppm fluoride-
A. Optimum and beneficial
B. too much
C. too low

33. most important property of major connector


A. Rigidity
B. Support
C. Balance
34. uncontrolled diabetes implication for implant failure

A. Inhibit osteoblastic activity


B. Promote fibroblast activity
C. Collagen formation

35. occlusal trauma not responsible for

A. Initiation of periodontitis
B. Contributing factor
DR. STRANGE

the only factor that can initiate periodontitis is plaque. All the rest are only contributing factors that can
make the problem worst

36. 13 Informed consent reflects which of the following ethical principles?


A. Justice
B. Autonomy
C. Beneficence

37.which is not for provider protection

A. Rubber dam
B. Single scoop technique
C. Recapping

38- pseudo epithelial serous lesion- granular cell tumor

39. geographic tongue

40- squamous cell carcinoma on floor of mouth

41. most useful in primary class 2 intracoronal restoration

A. Wide occlusal table


B. Wide contacts

42. implant overdenture-retention and support by which


Support by tissue
Retention by implant

43. fibroma, papilloma

44. compression rate -cpr

A. 80
B. 100
C. 120
adult is at least 2 inches which is 5 cm, all with a rate of 100-120 per minute.

45. red bacterial complex: PTT


Red complex is the bacterial complex responsible for causing perio disease which are .
P.gingivalis .
T.forsythia
T denticola

46. oral burning chronic: Candidiasis


melkerson
-candida

47.Cheek bite on a complete denture is due to ?


A. Insufficient horizontal posterior distance in upper and lower molars
DR. STRANGE

B. Insufficient vertical posterior in upper and lower molars overlap

48. what is not necessary for biohazard container


A. Made up of metal
B. Closable
C. Leak proof

49. if distance goes from 16 inch to 8 inch. Change in intensity


A. Double
B. 4 times
C. Half
D. One fourth

50. bundling unbundling


Bundling: same distinct procedure
Unbundling: Separate procedure

51. cleidocranial syndrome- supernumerary teeth


52. ectodermal

53. which is not differential diagnosis of Dentigerous Imperfecta.


A. enamel hypoplasia
B. ectodermal dysplasia

54.normal maxilla and retruded mandible

Sna 81 snb 68
Sna 86 snb 78

Marvelous
5th march rqs

1)High risk caries patient when u take X-ray

6 months
Before 6 months
6-18 months
3-4 months ?

2)contraindication of caoh
Mild cold sensitivity
Deep caries with pain since one month

3)patient with the history of bite on bread feeling sensitivity to cold


Cracked tooth
Vertical root fracture

4)pendulous tuberosity what u do


Osseous contouring
Remove it surgically

5)how you reduce mercury poisoning in environment

Recycling of mercury with amalgam separator


Use less mercury in amalgam
DR. STRANGE

6)acute hyperventilation
Vomiting
Carpopedal spasm

7)HBA1c
Detect blood sugar level
Detect glycosylated hemoglobin

8)all of the following can be seen in hypoglycemia except


Confusion
Bradycardia

9)irreversible pulpitis which pain


Quick sharp pain
Dull throbbing ache

10)diphenhydramine causes dry mouth by


Anticholinergic
Antihistamine
Antimuscarinic

11)patient came with the history of trauma which happened few weeks ago teeth have radiolucency what is ur next
plan
A. Do rct of all teeth
B. Recall patient and don't do anything
C. Do ept of all teeth n do rct of those which are not responding to ept

12.patient has gingival swelling in pic he has bruise on extremities what it can be
Gingival hyperplasia of drugs
Herpes
Leukemia

13.adjunctive to periodontal treatment all except


Fluoride
Tetracycline
Metronidazole
Chlorhexidine(peridex)

14.turner tooth
Trauma during pregnancy
Congenital syphilis
Infection

15.infected dentine and affected dentine what is correct


Infected dentine no need to remove
Affected dentine don't remineralize
Affected dentine n infected both have decalcification
Infected dentine have more bacteria

16.caries removal how


U remove caries from pulp 1st thn periphery
U remove caries from pulp only so no bacteria will remain
U remove caries from periphery thn pulp so that no bacteria accumulate at pulp

17.occlusal trauma causes


Widening of pdl
Vertical pockets
Horizontal bone loss
Vertical bone loss
DR. STRANGE

18.Horizontal root fracture of middle third n crown is slightly mobile


Don't treat it
Splinting it,n recall patient
Remove coronal portion n splint root portion

19.indirect temporary restoration causing periodontal disease


If not polished well
Margin kept short
Margins impinging in soft tissues

20.gingivoplasty can be done in


anug
Herpes
Pemphigoid

21.cleidocranial dysplasia common feature


Delayed eruption
Missing teeth

22.east west elevator is used for


Mandibular molar roots
Broken down palatal root of upper
Maxillary molar roots

23.anchorage is orthodontics except


Splinting two teeth together
Bracketing two teeth
Removable appliance with finger spring

24.most dangerous for dentist


HepB
HepC
HIV
Tb

25.keratosis is dysplastic if occurs on


Floor of mouth
Lateral border of tongue
Buccal mucosa

26.lateral periodontal flap contraindication


If inadequate donor tissue

27.which of the following has good prognosis


Carcinoma of lip
Adenocarcinoma
Carcinoma of floor of mouth

28.heavy duty gloves used for


When treating blood borne pathogens
When cleaning instruments

29.patient is anxious of dental treatment she feel helplessness bcz of anxiety what u do

Schedule small appointments


Ask her to raise hand when she feel anxious

29.cognitive coping
Outer
Inner

30.all can happen in shock except


DR. STRANGE

Hypertension
Tachycardia

31.all can happen in hypoglycemia except


Bradycardia
Confusion
Convulsion

32.fear of choking related to dental treatment


Needle phobia
Anxiety
Catastrophe

33.root caries what is true


I marked soft with in dentine something like this

34.Cleidocranial dysplasia what is correct


Delayed eruption
Hypodontia
Thin hair

35.Ectodermal dysplasia what is correct


Oligodontia
Supernumerary teeth

36.ectodermal dysplasia what is correct


Enamel hypoplasia
Hypodontia
Hyperdontia

37.kid in dental office u gave him La with epinephrine only one cartridge he felt agitated after giving la what do u
suspect
Epinephrine is more in la
Lidocaine toxicity

38.NO indication
Anxiety
If patient insist u for sedation
If u are going to do larger procedure

39.there was swelling 3*3 on the floor of the mouth n swelling is fluctuation soft . What will be the treatment
Incision and drainage
Marsupialization

40.Collimation does what


Reduce the intensity of beam
Restricts the dimension of beam (beam size)

41.Neuropraxia
Loss of axon n epineurium continuity
Discontinuity of axon and myelin sheath
Maintained continuity of axon n epineurium

42.2 year old child mother complaining that my son doesn't eat anything due to pain he has sore red gums what is
your diagnosis
Herpes
Eruption hematoma
Eruption cyst
Normal

43.patient complaining strain muscles n generalized soreness after denture she has also ulcer around corner of
mouth
DR. STRANGE

Inc vdo
Dec vdo

44.through and through furcation what u don't do


Tunnel
Guided tissue regeneration

45.patient came with the history of swelling and around mand premolar she has fever 101 and difficulty in sleeping
tooth has grade 3 mobility what u will do

Give antibiotic only


Refer to physician as lack of sleep
Extract tooth
Incision and drainage n recall

46.papilloma 2 cases
47.ptm
48.herpes 3 cases
49.amalgam tattoo
50.cellulitis treatment : Penicillin
Incision and drainage
Only antibiotics

51.patient with the history of bleeding n also bruising on extremities what it can be

Lichen planus
Leukemia
Medication

52.bonding is difficult to accomplish where


When u have not removed dentine during tooth prep
When sclerotic dentine u didn't remove it while cavity prep

53.moa of progesterone in gingival inflammation: increase vascular permeability and gingival edema.

54.moa of corticosteroid in asthma: Anti- inflammatory

55.moa of hyperbaric o2 therapy : Form new blood vessels- neovascularization and also collagen formation.

56.definition of emphysema

57.utility gloves : cleaning instrument


58.function of non rigid connector in fpd: flexibility for a path of insertion
59.methotrexate analog: Trimethoprim, Folic Acid.

60.cellulitis patient case what u will do 1st


Give antibiotics
Incision and drainage
Refer him to omfs

61.similarity between chronic perio n localized aggressive perio


Use of therapy
Local factors
Location

62.least complication of pulp necrosis


Concussion
Intrusion
Extrusion
Lateral luxation

63.levonordefrin and epinephrine act on which receptor


DR. STRANGE

Alpha 2
Alpha 1
Beta 2

64.which is more damaging to periodontium after restoration


Overhanging restoration
Gingival soft tissue
Damage of periodontium after tooth preparation

65.function of non rigid connector in fpd


Something related to non parallel abutment was true option

66.kid fell down from bicycle and he got injured.he has computing and headache what is your further management?

Refer him for medical evaluation because vomiting and headache is due to head injury

Don’t refer him because vomiting and headache is a common symptom after head injury

Wait for 24 hours then send him for medical evaluation

67.patient is going for open heart surgery next month he is hospitalized,cardiologist sent him for dental clearance he
has tooth#13,14 non restorable.tooth #14 has radiolucency what is your treatment plan

a)give him prophylaxis and extraction of both tooth


b)give him prophylaxis now and extract teeth after surgery
C)don’t do anything bcz it can lead to endocarditis
d)do rct of remaining roots of both teeth followed by coronectomy

68.when AAPD(American academy of pediatric dentistry) provide dental care something

6 months
12 months
After 18 months
After 24 months

69.methotrexate analogue
Folic acid
Leucocite

70.similar movements recorded in plane line articulate and semi adjustable articulators

Lateral
Protrusive opening closing
Opening closing
Only protrusive and lateral

71.kid came with pain in lower molar u gave him anesthesia 1 cartridge only after giving anesthesia he was agitated n
one more word they said (don’t remember)what happened to him?

Anesthesia was more


Lidocaine allergy or toxicity (don’t remember exact words)
Epinephrine toxicity

72.one case of lady around 40yrs old she has macular on tongue with 1.5*2cm n one more thing what it can be ?

Neurofibromatosis
Addison’s disease
Peutz jeghers

73.short case with histopath of nasolabial cyst they ask which cyst it can be
I marked nasolabial
DR. STRANGE

74.one case on opg radiolucency around one premolars small roots of one molar they asked brother has same thing
n kid was 12 year old
AI
DD
DI
Regional odontodysplasia

75.what type of resorption in ankylosed primary molar

Replacement
Ankylosis
Internal

Proscar act on which receptor : 5a-reductase inhibitor


Alpha1
Alpha2
Beta1
Beta2

77.which of the following drug act on centrally acting mu receptors

Flurbiprofen
Ibuprofen
Naproxen
Tramadol

78.where u give vertical releasing incision?


At the line angle of tooth
At middle third

79.regarding vertical releasing incision true/false: True


Vertical releasing incision should be given on line angle of tooth
Vertical releasing incision given for visibility and access

80.periodontal dressing provide all except


Minimize patient discomfort
To protect wound
Maintain tissue placement
Prevent plaque accumulation

81.one small case of patient smoker n he has red spots with white spots on palate what will be your diagnosis

Papillary hyperplasia
Chronic inflammation
Nicotinic stomatitis

82.purpose of bevelling in class 5

For esthetics
For good bonding
For larger surface area

83.least amount of reduction of pfm crown


2mm
1mm
1.5mm
0.5mm

84.one case in which patient has 3 lower incisors 2 of them has broad pulp chamber n canals separate. What it can
be?

Germination
DR. STRANGE

Fusion
Concrescence

85.do endo diagnosis well every single point is important from type of pain n fibers of pain to chronic abscess

86.one case of kid with clinical picture,he had swollen n red lips and asked what is your diagnosis

Herpes
Leukemia

87.patient wearing denture since 16 years she has loose denture now (same DD prostho case )

Ans was lack of posterior occlusion

88.patient on corticosteroids difficult to manage why

Cardiac arrhythmia
Hypotension
Increase Basal metabolic rate

89.cerebral palsy related to anterior tooth fracture

90.In which case u see pulp necrosis more

Apical abscess
Symptomatic apical periodontitis
Periodontal abscess

March 11
Posted by Dr. Purmina

1. Result not right but consistent what is that called?


type 1 or type 2 error
Answer: Reliability

2. Scientific article.. Where will you find details about study population ?? no answer sorry
Answer: Cross sectional study

3. .Teeth supraerupted opposing absent . if we will plan to replace opposing then what is the main reason for getting
supraerupted tooth in place ? didn’t get the question
Answer:

Q: A molar is supra erupted but has irreversible pulpitis, what do you do? : RCT & Crown.
4. examiner did study 3 man back for??? Didn’t understand the question
ANOVA
Analysis of variance is a collection of statistical models and their associated estimation procedures used to analyze
the differences among group means in a sample.

5. Bacteria least likely to be caused In


a. Perio surgery
b. Extraction and intentional replantation
c. Rct .. im toward this but not 100% sure?
d. Probing pocket

6.the similarity between simple hinge and semi adjustable ? didn’t find answer for this
simple hinge only maximal opening .. semi adjustable lateral excursion, accept face bow
DR. STRANGE

7. Methotrexate analogue?-FOLIC ACID

8. There was one in which there was a thin opaque line in the same direction of external oblique ridge: internal
oblique ridge or mylohyoid ridge

9. Why mucous cyst appears blue


A. The keratin inside reflects blue
B. All vascular lesions are blue
C. The mucoid inside reflects blue
10. if the whole maxilla comes out detached from other bones, what kind of fracture.
Lefort 1
Lefort 2
Lefort 3
Lefort 4

11. Which movement cant relapse most


Intrusion
Extrusion
Tipping
Rotation

12. Same 16 old kid…. Open bite, mamelons, put his tongue against his teeth, has a habit to cover his upper teeth
with his upper lip…. Why his open bite
Answer: Tongue thrust

13.16 old boy lost #8 in an accident…# 7 has a root canal by root is kind of short….What could happened with # 7?
A. Removed apex before reimplant
B. apexification
C. Surgical apex with endo….
D. Other 2 options

14. Nistatin in HIV patient- False Nystatin is for Candida

15. old Amalgam on 12…mod, gray-bluish stain and JUST in occlusal view, what do you see?
a. Amalgam tattoo
b. Recurrent decay

16. Amalgam MO how can fix it?


A. Narrow isthmus
B. Wider Isthmus
C. Grooves
DR. STRANGE

17. GIC
A. High tensile
B. Low tensile

18. Dentist give anesth. To a boy in pain ( IAV) and didn’t work, and decided do :
A. anesth buccal
B. pulp directly

19. Which muscle is SPAM in non working movement


A.Masseter
B. lateral
C. medial
D. temporal

20. A pt taking digitalis for CHF the question has something to do with fluid elimination? Digi increase pumping,
diuretics get flow out. Increase contractile force of cardiac muscle

21. What is seen in symphysis synchondrosis of the mandible? it is a not true symphysis as there is not
Cartilage networked two sides of the mandible .

22. What is least important when selecting teeth to be used for overdenture?
A. iimp is amount supportive bone
B. number exist roots
C. type and amount occlusal force
D. type attachment,splint or non splint roots

23. When doing a gingivectomy the internal bevel should be placed?


3 mm coronal to MGJ 1-2 mm from gingival margins t/f: TRUE

24. Inverse square law? Moved from 12 to 4 feet is 9X

25. LED curing light vs another aren’t able to cure all resins because they can only cure within spectrum? BOTH
TRUE

26.Operculum on tooth #18, how should you treat? Tissue out of occlusion, ext, adjust #15, scale under
operculum

27. Chi square and when to do? for


-Non parametrical values
-two common variable
-two categorical variable
-determine if the observed frequency are different form what we would expect

28. Patient has untreated chemical burn at corner of mouth that has healed, What are ramifications?
A. Lingual dentition
B. labial displacement
C. Decrease Vertical Dimension?

29. What is the advantage of metal framework over plastic? Except type question
30. Most often cause of failure of amalgam restoration?
a) condensation not strong enough
b) moisture contamination
c) too long trituration
DR. STRANGE

30. Where is NOT recommended to do gingival graft – which site?


a). Premolar
b). Molar
c). CANINE area
31. Dr.Whitaker class - db cusp occlude where in opposite side
max DB cusp to mandi distobuccal groove
mandi DB to max Central fossa
32. Xerostomia-opioids does not cause dry mouth; sjogren's syndrome does cause arthritis TRUE
33. Everything is INITIAL Perio treatment Except:
A. OHI
B. Calculus Scaling
C. Root Surface Planing
D. Antibiotic
34. Kid is very skinny, thin, pale, long nails what is disease ? Ectodermal derive structures
35. Patient is in your office with infection/inflammation on nose, cheek, eye. You must t identify infection – is it
Odontogenic or not? NON ODONTOGENIC

36. You are given a PAN with wide black stripe (almost a third of PAN) on your Right and on you right side you can
see a spine of patient. Middle of PAN is normal. How can you describe defect / error of PAN?
A. Is it an open mouth
B. Was film too long in developer or fixer
C. Was the door open during development
D. Was machine stuck during PAN rotation (upon pt’s shoulder)?
37. radiograph with radiolucency at bottom, didn't say patient had an extraction or anything: benign neoplasm cyst,
odontogenic cyst, salivary gland duct cyst...??
38. Pt. presents to the office and complains of a discolored crown on t#8. You notice that the patient needs perio
treatment. The patient only wants the crown replace. WWUD

39. Patient asked about eyes, gloves masks, what don’t you say don’t worry we have it under control, universal
precaution
40. How do you test a perm tooth with an immature apex hot test --FALSE
41. Teenager with proximal decay that will undermine all cusps. What restoration will u do on it? CROWN
42. Rate of epithelial junction regeneration after periodontal surgery is 0.5-1.0 mm, 5-14 days. TRUE
43. HIV viral load of 100,000 T-cell count of 30 means - you have a high viral load and are susceptible to infections.
Pt’s T cell count is too low. NO TREATMENT too low.
44. How is the junctional epithelium attached to the tooth? Hemidesmosome

45. Which tooth is most likely to be extracted if affected by periodontal involvement? Max Molars.
46. pt. had a fear of “stroking out” in the dental chair while receiving care…how would you treat them?
47. Bleeding time is involved with: von willebrand’s
disease
48. Class V on Canine. how do u bevel incisal and cervical
margin.
cavo superficial margin for composite restoration is beveled
when placed on enamel ( major difference with amalgam).
This beveled is etched and provides retention for the
restorative material, improves marginal seal and mantenía
resin’s strength with enough bulk.

49. xray 4yr old u don’t see 2nd pm do you


A. refer to pedo
B. close space
C. maintain space
DR. STRANGE

50. lesion on tongue the same exact lesion on palate what is it?
A. Candidiasis
B. Syphilis
C. lichen planus
51. Main complication during RCT-
A. ledge formation?
B. perforation
C. instrument separation
D. vertical root fracture

52. Pedo mgmt- sedations, behavioral mgmt


53. If trying to take max impression and access buccal space, what muscle would be in the way. I put masseter,
maybe orbicularis oris. Other choices were med and lat pterygoid..BUCCINATOR?

54. What you see to differentiate b/w acute perio abscess and acute periodontitis, how to differentiate b/w chronic
and periodontal abscesses
Abscess apical chronic: necrotic pulp
Periodontal abscess: vital

55.it asked me that since this patient is on cocaine should I not expect him to maintain his
oral health. True

Rawa MARCH 13, 2019

My exam was 50% req


10-15%new .weird .unbelievable.
The rest was basic ...from dd .

1.pat with liver dysfunction .what LA...all options was Amide: ARTICAINE

2.what medicine would u give pat with Sjogren's Syndrome to relieve hyposalivation.

A. cholinergic agonist (treat myasthenia gravis)


B. Cevimeline (can treat dry mouth caused by sjogren's Syndrome)
C. pilocarpine (treat dry mouth sjogren's and glaucoma)

3. HPV that cause oropharyngeal cancer affect what ....I chose cytochrome P53

4.what strain of HPV cause oropharyngeal ca .


a.11
B.16
C.18
D.13.

5. Most common perio disease in school age children. Marginal


6. LD50..definition.: LEthal Dose

7.Temporary facial palsy occur after injection LA .


a too posterior
b.too anterior
c.too high
DR. STRANGE

d.too low .

8. Combination syndrome read everything I got 3 Qs.

Combination syndrome: refers to what is believed to be a specific pattern of bone resorption in the anterior portion
of an edentulous maxilla, caused by wearing a complete denture opposing natural anterior teeth

9. Know what is Iatrogenic and what is not.


10.what will allow complete passage of transillumination
a craze line
b.split tooth
c.crown root fracture

11.percussion is to evaluate: PDL

Q. Which teeth do you perform pulp evaluation on?


A. Tooth only
B. Tooth and neighboring tooth
C. Tooth, neighboring teeth, contralateral tooth
D. tooth, neighboring teeth, opposing teeth

12. least tooth involved with fracture .


a maxi molars
b.mand .molars
c.maxi premolars
d.mand .premolars.

13. Conscious sedation what reflex should keep : vibrate

14. Montelukast MOA: Montelukast is competitively and selectively antagonizes CysLT1 receptor (leukotrienes
receptor) mediated Bronchoconstriction.Montelukast inhibits physiologic actions of LTD 4 at the CysLT1
receptor without any agonist activity.

15. Anaesthesia of lower lip indicate..malignancy


16 .High C factor seen in what type of restoration ?...I chose class I.
17. avulsed primary central incisor ..keep it out.
18.what type of oral cancer is most common in USA

a. lip
b. Tongue
c. BCC

20.Ecchymosis of floor of the mouth after trauma on mandible indicate which site fracture?: Mandibular body or
symphysis fracture

21.the most important factor to reduce sensitivity of root after perio treatment is ..plaque removal .

22. Infection to which space will present as swelling clinically .


A. Canine space .
B.pterygopalatine .
C.retropharyngeal.
D.Infratemporal.
DR. STRANGE

23. The facial bow in maxillary retainer used to .


A.tipping maxi teeth lingually .
B.produce no movement .just retain .

24 . When can the dentist take x ray for 4 years old child caries free clinically? Molar has close contact

25.difference between gingival margin trimmer and hatched.

26. Clubbing fingernails in children indicated


A.thumb sucking B.cyanotic heart problem.

27.Patient complain light colour of anterior filling what should u do ( there were not tint in the options).
A.add the adjacent color from the wheel
B. add the counterparts color from the wheel
C .add glycol blue ( no idea what is this )

28. .pat has lesion on posterior palate need biopsy ; what nerve should get infiltration LA.
A.long buccal.
B greater Palpatine
C.posterior superior alveolar.

29.pat taking methotrexate for RA; what concern should the dentist take ? ..thrombocytopenia ..not sure ..but the
most reliable

30. .what epithelium is found at the base of sulcus ?? I got junctional epithelium in the options .

31. .function of the key way in post preparation. Anti-rotation

32. .high risk to get retinal damage from dental staff.


Answer: Dental Assistant

33. Most common pattern of osseous defect in chronic periodontitis ? .not horizontal .I went with cutare.

34.which teeth can be removed without pain when you give IAN block and lingual nerve block ? Premolars & Canine

35..why Gold made alloyed??


A.to increase strength .
B to increase elongation .

36.function of Saline in composite restoration .?? No coupling agent in options . Adhesive?

37 .what element added to alloy to produce corrosion resistance?


A.nickel
B.cobalt .
C.chromium

38.why mucocele looks blue clinically? Vascularity

39.in what order the 3 canals of mand 1st molar will appear?
2 Canal in distal root
DR. STRANGE

40.small amount of metal oxidized added to provide what??I think the oxide layer in pfm crown !!!

41.implant analogue ..what is it for? All options have something with final impression ..so read it well .

42.combination clasp?

43..clinical attachment loss .learn how to calculate it.

44.what systemic condition associated with endocarditis and glomerulonephritis ?? SLE.

45. .primary support for mand CD? Buccal shelf .

46.what is not important in class 3 cavity preparation?? Extension for prevention .

47.x ray shows very small caries lesion cross the DEJ on mesial and distal surface of tooth #3??? DO and MO
separate

48.function of the filter in x ray machine?

49.anterior tooth has RCT done and retrograde filling with amalgam shown in x ray. why it appears gray?
A .initial trauma
B.from retrograde amalgam or
Necrotic pulp

50. pat 14 years old girl .has no plaque but she has 6 mm attachment loss on both teeth#3&#14.what bacteria ?

51.Epulis fissuratum all except antibiotics .

52. disadvantage of autograft Cancellous .to be placed in mandible .

53.most recurrent caries after MO composite filling ? Gingival

a lot of qs about stress related to pain and anxiety ...I felt I should read something about psychology .!!!
Read about clasp ..few questions about position .why it get broke in function .

54.occlusal trauma causes


Widening of pdl
Vertical pockets
Horizontal bone loss
Vertical bone loss

55.Horizontal root fracture of middle third n crown is slightly mobile


Don't treat it
Splinting it,n recall patient
Remove coronal portion n splint root portion

56.indirect temporary restoration causing periodontal disease


If not polished well
Margin kept short
Margins impinging in soft tissues

57.gingivoplasty can be done in


DR. STRANGE

anug
Herpes
Pemphigoid

58.cleidocranial dysplasia common feature


Delayed eruption
Missing teeth

59.east west elevator is used for


Mandibular molar roots
Broken down palatal root of upper
Maxillary molar roots

60.anchorage is orthodontics except


Splinting two teeth together
Bracketing two teeth
Removable appliance with finger spring

61.most dangerous for dentist


HepB
HepC
HIV
Tb

62.keratosis is dysplastic if occurs on


Floor of mouth
Lateral border of tongue
Buccal mucosa

63.lateral periodontal flap contraindication


If inadequate donor tissue

64.which of the following has good prognosis


Carcinoma of lip
Adenocarcinoma
Carcinoma of floor of mouth

65.heavy duty gloves used for


When treating blood borne pathogens
When cleaning instruments

66.patient is anxious of dental treatment she feel helplessness bcz of anxiety what u do
Schedule small appointments
Ask her to raise hand when she feel anxious

67.cognitive coping
Outer
Inner

68.all can happen in shock except


Hypertension
Tachycardia
DR. STRANGE

69.all can happen in hypoglycemia except


Bradycardia
Confusion
Convulsion

79.fear of choking related to dental treatment


Needle phobia
Anxiety
Catastrophizing

80.root caries what is true


I marked soft with in dentine something like this

81.Cleidocranial dysplasia what is correct


Delayed eruption
.Hypodontia
Thin hair

82.Ectodermal dysplasia what is correct


Oligodontia
Supernumerary teeth

81.ectodermal dysplasia what is correct


Enamel hypoplasia
Hypodontia
Hyperdontia

82.kid in dental office u gave him La with epinephrine only one cartridge he felt agitated after giving la what do u
suspect
Epinephrine is more in la
Lidocaine toxicity

38.NO indication
Anxiety
If patient insist u for sedation
If u are going to do larger procedure

39.there was swelling 3*3 on the floor of the mouth n swelling is fluctuation soft
What will be the treatment
Incision & drainage
Marsupialization

40.Collimation does what


Reduce the intensity of beam
Restricts the dimension of beam

41.Neuropraxia
Loss of axon & epineurium continuity
Discontinuity of axon & myelin sheath
Maintained continuity of axon n epineurium
DR. STRANGE

42.2 year old child mother complaining that my son doesn't eat anything due to pain he has sore red gums what is
your diagnosis
Herpes
Eruption hematoma
Eruption cyst
Normal

43.patient complaining strain muscles n generalized soreness after denture she has also ulcer around corner of
mouth
Decrease VDO

44.through and through furcation what u don't do


Tunnel
Guided tissue regeneration

45.patient came with the history of swelling and around mand premolar she has fever 101 and difficulty in sleeping
tooth has grade 3 mobility what u will do
Give antibiotic only
Refer to physician as lack of sleep
Extract tooth
Incision and drainage n recall

50.cellulitis treatment
Incision and drainage
Only antibiotics

51.patient with the history of bleeding n also bruising on extremities what it can be
Lichen planus
Leukemia
Medication

52.bonding is difficult to accomplish where


When u have not removed dentine during tooth prep
When sclerotic dentine u didn't remove

March 19, 2019


Sumaya

1. primary tooth with pulp necrosis what's the treatment : no pulpectomy in options , but extraction there
ANSWER: EXT

2. patient with warfarin with inr 2.1 how to manage?


ANSWER: Normal

In healthy people an INR of 1.1 or below is considered normal. An INR range of 2.0 to 3.0 is generally an effective
therapeutic range for people taking warfarin for disorders such as atrial fibrillation or a blood clot in the leg or lung.

3. flap for tori


ANSWER: Double Y

The most commonly used incision design is a “double-Y.” A midline incision is made anteroposteriorly from a point
several millimeters anterior to the margin of the torus. This full-thickness incision is carried to bone posteriorly until it
DR. STRANGE

reaches the most posterior visible point on the torus, or a point approximately a centimeter anterior to the hard-soft
palate junction. Care should be taken to leave room for posterior releasing incisions to extend obliquely in a lateral
and posterior direction from the posterior end of the midline incision without violating the soft palate. Completing
these posterior releases is sometimes easier after the bulk of a large torus has been exposed or even removed. At
the anterior end of the midline incision, oblique releasing incisions are extended laterally and anteriorly to end lateral
to the lateral margins of the torus.

4. signs of acute MI
-pressure or tightness in the chest
-pain in the chest, back, jaw, and other areas of the upper body that lasts more than a few minutes or that goes away
and comes back
-shortness of breath
-sweating
-nausea
-vomiting
-anxiety
-a cough
-dizziness
-a fast heart rate

5. nikloky’s where
Answer: Pemphigus , pemphigoid , erythema multiforme, epidermolysis bullosa

6. patient with heroine what's the pain medication c.i


ANSWER: Heroin metabolized by Liver so we have to avoid acetaminophen and can prescribe NSAID as pain
medication.

Morphine for narcotic addiction

7. plavix how to manage


Answer: NO need to stop it before procedure.

8 permanent first molar extraction for kid 8 y old what's next step
A. Space maintainer
B. wait second molar to erupt

9. all defect for midline except


A. cleft lip
B. tongue
C. thyroid

10. which tooth susceptible to pulp exposure in class 2 preparation


Answer: mandibular 1st premolar

11. blood supply for muscle of mastication


Answer: Maxillary artery
12. most common salivary gland have tumor

Answer: Parotid tumor/Pleomorphic is most benign and it's the most common type of salivary tumor. 80%

13 preliminary school the student have high interproximal caries


DR. STRANGE

What's the program help to Dec the caries :


A. Fluoride
B. sealant
C. diet

14. most common place has abscess in primary teeth


15. case have multiple KOT and nodular on the skin what she has ?
Answer: gorlin-goltz syndrome

16. type of tooth fracture incisal edge but supragingival


Answer: Class I

17. less Likely to occurs during extra upper first molar there is
A. condylar fracture
B. osteitis

18. cause of adrenal atrophy

Primary adrenal insufficiency is caused by the destruction of the adrenal gland. Idiopathic adrenal atrophy is the
most common cause of adrenal insufficiency. It is not known exactly why this occurs, but it is believed to be related to
an autoimmune response that results in the slow destruction of adrenal tissue. Tuberculosis, histoplasmosis, AIDS,
malignancies, and hemorrhage into the adrenal glands have all been associated with destruction of the adrenal
glands. All patients with adrenal insufficiency or steroid-dependent disorders are at risk for an acute adrenal
crisis.
Other causes include physiological stress, including surgery, anesthesia, fluid volume loss, trauma, asthma,
hypothermia, alcohol abuse, myocardial infarction, fever, hypoglycemia, pain, and depression.

19. retention of post all except length width cement


20. what u doing when extra primary tooth and very small fragment of root fracture inside: Leave it

Deicy
March 18, 2018

1-3 molar extraction indication


To avoid caries in the adyacente tooth
Because of bone lesion in the tooth

2-What is hybrid layer:


Composed by Removed organic component of dentin including collagen and monomers inside of this space
polymerized (I don’t remember clear other options but this is the answer)

3-Epulis is histologically similar to what:


Traumatic fibroma
Traumatic neuroma
Granular cell tumor

4-Most missed tooth/


Maxillary Lateral only sane option
Mandibular first premolar
Maxillary first premolar

5-Lingual varices:
DR. STRANGE

age (I marked age)


hypertension

6-Why risk of bleeding during chemotherapy:


Thrombocytopenia (I marked this)
fragile vessels

7-Patient has been in IV chemotherapeutic as unrestorable tooth


- Endo
- Suspend 3 month and exodontia

8-Cleft lip
6-9 weeks only same option
2-3 weeks
14 -16

9- a study failed to to recognize people without the disease


false positive (I always have doubts with this kind of qs, check this answer not sure)
false negative
positive predictive
negative predictive

10- the anterior study lack of


sensitivity
specificity
reliability
validity

11-Qs about scaler, its main mechanism of actions is


It’s better effect is because is sharp point
Sharp edges
Vibration
Pressure over the teeth

12- probing depth 2, gingival margin is 1 mm below of CEJ


3
2
4
1

13-Overdenture
Retention Implant /Support tissue (other combinations)

14-Which one is not part of the “R” for smoking cessation


Repeat
Reward
15-Ear lobe on x ray (really easy to identify nothing strange)

16- Pathology that causes skull enlargement


seizures
Hydrocephalus
Other options I can’t recall
DR. STRANGE

17-The less reliable wait to measure pulp status


Etp
Cold
Percussion (this is for ligament inflammation)

19 EPT and fibers stimulated


A delta
C fibers
other options

20- retention of sealants is caused by


Tags in enamel to create mechanical retention
Chemical bond between sealant and enamel

21-Related to soft tissues, Modified pontic should:


light touch in soft tissue
it should make strong pressure on tissue
model should be scrapped if the pontic is made in gold
model should be scrapped if the pontic is made in metal ceramic

22-Most harder to clean:


ovate
modified ridge
conical
hygienic

23-Most probably to develop neurofibrosarcomas


neurofibromatous (I marked this)
traumatic neuroma
neurilemoma

24-tumor that has Enamel dentine and cement:


odontoma (I marked this, I was confused between this and second option)
ameloblastic fibro odontoma
dentigerous
other option

25- Qs about Cleido:


Supernumerary (was one of the answer)
delayed eruption (second answer)

26-Upper premolar forceps: 150

27- advantages of GIC over composite


fluoride release and chemical bond

28-Lido calculation in grams


articaine 1.7 at 3%
3.4mg
0.34mg
34mg ( this is the answer, unfortunately I marked 3,4 mg ☹)
DR. STRANGE

29-fibrous dysplasia x ray

30- skin nodules, supernumerary teeth we should be worried about:


intestinal polyps (Garner)

31 which disease is most probably to develop osteosarcoma -Paget

32-What no DD in dentin dysplasia- Enamel hypoplasia

33- What cause less bone loss:


leukemia
oral hygiene (other options for sure were not the answer)

34- gingivectomy what is true


Is incision is made slightly apical to the pocket base.
Incision should be made just coronal at mucogingival J

35-Lateral flap indications


to increased attached gingiva
osseous contour
SRP

36 2 qs about distance in between implant 3mm

37 new study found relation between Asthma: leukotrienes

39- less probably to cause ledge:


Short canals
Long
Curve

40- 2 qs about Antagonist: high.affinity.... LoW intrínsic activity

41- Naloxone MOA


high affinity high intrinsic activity
high affinity low intrinsic activity
low affinity and low intrinsic activity
low affinity and high intrinsic activity

43- diazepam overdose. flumazenil

44- MOA of clonidine except


-Option were really complicated but as soon as a saw
Central acting alpha blocker I knew this was the answer (because it is alpha 2 adrenergic agonist)

45-Patient with ADHD is most probably taking:


Amphetamines
46-% fluoride water access in USA:
74%

47- what is NOT the main cause that teeth of complete denture fall out:
excessive separator medium in the dental flask before pouring the acrylic
DR. STRANGE

inadequate wax removing ,


cooling and heating processes (I choose this option because at least for me it is the most reasonable answer)
teeth that are not made of PMMA

46-Mínimal width for implant FL:


7
4
9
12

47-. When taking an intermaxillary bite registration at centric, after using an arbitrary face bow, there must be only 1
mm separation on second molar area. Why:
A. was must always be 1 mm thick when using an arbitrary face bow
B. thin records cause minimal distortion

48-. Bite registration on centric must be:


A. thin just the cusp tips without perforations
B. think but a few perforations are OK
C. thick

49-if a file broke in apical third which condition have the better prognosis
vital pulp and not periapical lesion
necrotic pulp with not periapical lesion
acute apical lesion

50-what is the main purpose of pulpotomy :


allowing biological formation of the root in open apex

51- if patient has personal relationship with patient what ethic code is violating: non-maleficence

52- refer to specialist: nonmaleficence

53- confidentiality of patient records: autonomy

54-modeling

55 for gag reflex: desensitization

56- it was created in 1997 to help child: CHIP

57-cause of cervical discoloration in anterior veneers that were cemented with light cured resin cement
amines (only it is cause if the cement is chemical cured)
break dawn of silane (silane is not related with discoloration)
no enough cement in cervical margin (only same option)
micro cracks in veneers (the discoloration is in the cracks not just in the cervical margin)

58-Opaque color coming through restauration, in incisal third what’s the problem? abutment under prepped in
second plane
abutment under prepared in primary plane

59 nitrous oxide does not cause: I forgot the option but the answer was
hypertension
DR. STRANGE

malignant hyperthermia

60 Nitrous oxide contraindication


depression
chronic pain
psychosis

61- which one is the less common treated with concomitant antibiotics
localized aggressive P
generalized aggressive P
abscess
SRP

62-about periodontal reevaluation what is true


Nerve it should be charged
Visits Every 3 months cause that patients have less compliance
Visits every 3 months helps to patients have more compliance

63- treatment for status epilepticus; Nit diazepam in options: Midazolam

64-Tuberculosis is mouth: features


Ulcers in tonsils and other places
I choose this option along with other places but not sure about

65-Histological changes in size and form


Histodifferentiation
Morphodifferentiation
Morphodifferentiation leads to increase in size & shape of the enamel organ to exhibit a bell shape
I think histo lead to type like ameloblast , cementoblast

66-For clinical trials in patient what it the most important (or first thing something like that)
-Informs consent to the patient
-Be sure the patient won’t be on risk
67- After survey what is the next step
-prep rest
-prep indirect retention
-option really strange about retention ,,, even though it was a little bit strange was the only same option

68- patient in Plavix and baby aspirin (81mg) have increase bleeding time
patient needs to stop medication before surgery

Both true
Both false
First true, second false
First false, second true

69- LED photocure devices are no able to light-cure some composites, this is because some composites have photo
initiators that are not in the range of cure that leds have.

Both true
Both false
First true, second false
DR. STRANGE

First false, second true

70-HPV
before 18 years was in the options

71% percentage of herpes prevalence in USA


80-85% was in the options

71- picture of a person with ulcers in the lips and checks just until the midline, with picture and what the patient states
you can conclude
-Herpes zoster

72- Hairy leukoplakia cause mainly by


HIV
HPV
EBV

73- is the distance change from 8 to 16 what happen with the intensity:
¼ of the intensity
4 times intensity
double intensity

74- main function of the collimator (2 qs bout collimator, second one was a little bit strange but if I am not wrong the
answer was something about decreasing amount of radiation, confusing options)
control beam size (something like that)

75-pic of papilloma in tongue


76- picture of a patient with asymptomatic bump in the palate (I looked like torus but most to the left side) which one
is not a differential diagnosis
periapical abscess ( I choose this I am not sure about)
torus
lymphoma
pleomorphic adenoma

77- most common lymphoma in jaw burkitt’s lymphoma was in options


78- adrenal crisis qs: hypotension
79. description of fever, high blood pressure, agitation.: thyroid storm

80 – qs about which hormone can interact with epinephrine: thyroid, grave’s disease

81- Side effect of nitroglycerin. orthostatic hypotension, headache and something else
2 options not related (hypertension) but one more really close to the first one.

83- there was a case when we can chose the least possible treatment plan for him
● The answer was the treatment that propose joint natural tooth and implants

84-First thing to do in reevaluation appointment: Update medical history

85- what determine the extension of the occlusion of the occlusal plane
inter tuberosity distance
something with fox plane
centric relation
DR. STRANGE

2/3 the height of the retromolar pad

86- posterior palatal seal, what have to be evaluated (confusing and closing options)
vibrating line, throat form, resilient tissue something like that
(other option included patient age or something else, but the only one that had vibrating line was that)

87- in vestibuloplasty what muscle has to be medicated


buccinators
masseter
orbicularis oris

88- facial frenum in lower mandible. triangularis

89- Lisinopril what is not a side effect:


hypotension
hypokalemia (actually if I am not wrong it causes hyperkalemia)
interaction with long term Nsaids
dry nonproductive cough

90- pedal edema and something else


CHF
Angina
MI
Hypertension

91- side effect opioids ( yessss again lol)


productive caught
soft stool
mydriasis
XEROSTOMIA (ONLY SAmE OPTION)

92- Opioid toxicity triad


miosis, respiratory depression and coma
miosis, respiratory depression and hypotension-death
miosis, respiratory depression and seizures

93-there was a case with posterior cross bite (only one side) how to corrected
buccal inclination of premolar and molar of the affected area
lingual inclination if the lower teeth
other option not related
I marked the first one because it was clear the palatal inclination of the premolar and molar of the upper teeth)

94- Picture with fluorosis


95- Patient without many teeth they ask what is the less probable reason why the patient lost his teeth
congenial missed and malformation something like that: which is not true cause the alveolar bone was evident in the
x ray)
96- patient taking sertraline what we have to be worried about: epinephrine was one answer
97- with the same patient xerostomia caused by: medicaments

98-patient which anterior teeth are touching while doing sibilant sounds. What is the reason
lower anterior teeth too facial
upper anterior teeth too lingual
DR. STRANGE

increased of the vertical dimension ( I went with this)

99- what is dependent on the patient


postural VD or rest position
Vertical dimension
Centric relation blab la bla

100- patient with vertical dimension decreased and angular cheilitis treatment nystatin

101- patient with lesion in tongue what is not a differential diagnosis. nicotinic stomatitis

102 – feature that have materials for border modeling. thermoplastic

103-burning mouth syndrome related or caused I don’t remember


candidiasis ( I marked this )
stevens johnson

104-patient with almost all teeth but very bad oral hygiene. Dentist decide to present a treatment plan extracting all
their teeth assuming that the patient is not going to improve this habits he is violating which code
autonomy
non maleficence
beneficence
veracity

I market autonomy but I was not sure about it


105 patient with a couple of teeth with a little bit of dental plaque that needs a immediate denture what is the first step
SRP
Obtain a model ( I marked this, I don’t think it is reasonable SRP is the teeth are going to be extracted)
Extraction of teeth
Other option

106 what is not sure about post-operative in complete denture


patient should try to use all the time for the first 48 hours
patient may need a rebase in 9 to 10 months
reline it is always necessary (I went with this not sure)

107 after extraction patient is not able to open mouth, what could be the muscle implicated
external pterygoid
masseter
buccinators
medial pterygoid (I market this cause I think it is trismus after anesthesia )

108 after IAN LA patients face become inmovil in the same side what is the reason, anesthesia went too
posterior
anterior
lateral

109 palate biopsy what nerve has to anesthetized. greater palatine

110 after biopsy in the palate was found, keratinized epithelium, underline CT and cortical and medullary bone,
what’s is the most probably diagnosis
pleomorphic adenoma
DR. STRANGE

osteoma ( it is same as torus)


other options not related

111- after alveolar osteitis what it is the best treatment


antibiotics
sedative dressing
other options

112. 2 years’ kid came for checking but it is complicated something like that what is the proper management
-papoose board
-ask to auxiliary to restrain the kid
-ask to his mom for helping to restrain the kid

113. SlOB rule qs about the apparition of the canal of the first molar when x ray Is taken from mesial
palatine- mesio-buccal 2- mesio-buccal 1 –mesial

114 # of cartridge of 3% ( I am not sure) and 190 pounds person: I did the formula and the answer was 390mg but
since we cannot pass of 300 mg I chose 8

115- hyperventilation can cause


low O2 saturation
tingling fingers ( I went with this, even though first option was really tempting for me

116- patient: wheezing in expiration, hypotension, confusion ….. after anesthesia. anaphylactic
117 patients which salbutamol, and respiration problems, he can’t walk without stop after 10 steps. What he is not
going to be able to resist in the office
-LA with epinephrine
- Horizontal position in the dental chair ( I chose this cause his problems were related to the respiration not about
blood pressure)
118 in emphysema the the main problem is in:
chronic bronchial inflammation
pleura
lung alveoli

Read careful endo diagnosis, there are many qs on this topic. I didn’t get any qs about ANB or other angles, just one
case angle classification. MY test was mainly endo and perio and prostho.

March 20, 2019

1. Pain on neck , ear what disease?

2. Lesion on tongue from 2 weeks what to do next ?

3. Not an action of benzodiazepines? Antiemetic

4. Which muscle help in centric relation ?

5. Disadvantage of oral drugs on IV drugs.?

6. 1st sign of syncope ?

7. Not in ortho consent form ?

8. Moa of benzo ? GABA


DR. STRANGE

9. Point A ? Deepest portion of Maxilla

10. Hepatitis vaccine when ?

11. Which Leukemia cause gingival bleeding?

12. Perio disease related to what ? Cardiac murmur , endocrine problems

13. Hpv with scc site ?

14. Antagonist action ?

15. GIC properties over rgic

16. Green complex bacterias except ?

17. Caries caused by ?

18. Drug classification class 2 except?

19. Hyoid bone xray

20. Cementoblastoma x ray

21. Nutrient canals

Do perio well,there were big statements asking to choose if 1st true , 2nd false and are they related to each other

March 22, 2019


NBDE

1. Aneurysmal bone: Answer: PSEUDOCYSTS

Not true cysts- they are called : pseudocysts". ex. latent bone cyst, lingual mandibular concavity & aneurysmal bone
cyst.

2. Traumatic neuroma: Trauma to a peripheral nerve. Most common site over MENTAL FORAMEN in edentulous
mouths.

3. AOT (Adenomatoid Odontogenic Tumor): Teens; females; Anterior Jaw; in association with the crowns of impacted
teeth. Well circumscribed unilocular RL lesion, may have small RO foci.
DR. STRANGE

4. Saturation/ value
Saturation is chroma ..
Value is lightness and darkness
chroma increases with age
value decreases with age
hue doesn’t change

5. Flaps -indication of gingivectomy


-Elimination of Suprabony pockets, regardless of their depth, if the pocket wall is fibrous and firm.
-Elimination of Gingival enlargements
-Elimination of suprabony periodontal abscesses.
Important: An adequate amount of attached gingiva must be present before a gingivectomy is done; otherwise the
result will be an area with minimal or no attached gingiva.

6. Basic drugs for angina, antibiotics and anesthesia

Angina: Nitroglycerin. Acute -Metoprolol; Chronic- Atenolol, Calcium blocker (nifedipine) & Propanolol (Beta
blockers)

Antibiotics
Anesthesia

7. Increase incidence of what caries has increase- Answer: Root Caries


8. Major connector provides- rigidity
They should be Free of movable tissues
Relief should be provided
Not impinge on gingival tissue
Bony and soft tissue prominence should be avoided during placement and removal.

9. Radiation causes caries on what surface: answer: Cervical


10. Please review all the features of reversible pulpitis/ apical periodontitis:

Reversible pulpitis- Exaggerated response to THERMAL stimulus but once the stimulus is removed, the discomfort
DOES NOT LINGER. EPT (responsive).

Apical Periodontitis- RAPID onset, SPONTANEOUS PAIN, PUS FORMATION and often associated tissues.
Percussion (exquisitely sensitive)

11. Sliding board-wheelchair transfer: Answer: patient preference first


DR. STRANGE

12. Metamerism- is important in MATCHING SHADES of a metal-ceramic crown to a natural tooth. Color matching
should be done under 2 or more different light sources, one of which should be sunlight.

13. Desensitization. Answer: Gradual exposure, for fear.


14. Ian nerve block not effective due to what accessory nerve. Answer: Mylohyoid Nerve
15. X-ray with nutrient canal:

16. NO- not with nasal congestion


nasal congestion
copd
cystic fibrosis
first trimester of pregnancy
17.Nitrous oxide- side effect- Nausea
18. Periodontitis-black male
19. Sequence of tooth eruption primary- ABDCE
20. Parkinsonism antibiotic-
21. Dysplasia- carcinoma in situ- erythroplakia
Almost all true erythroplakia exhibit a microscopic picture of epithelial dysplasia, carcinoma in situ, or invasive
squamous cell carcinoma. Biopsy is mandatory.
22. Malignancy- paresthesia, Lower Lip, SCC
23. TMD - type a personality , type 2 occlusion,
24. Cracked tooth syndrome- Craze line, Transillumination
25. Amalgam tattoo- bluish black

26. Periodontitis-black male


27. X-rays of Paget- cotton wool, high level of serum alkaline phosphatase
Hypercementosis,headache, hearing loss, enlarge cranium, bone of increased density. Also known as “Osteitis
deformans”.
DR. STRANGE

28. Tension side: On the side of the roof from which tooth moves, you will find “OSTEOBLASTIC ACTIVITY” that
results in bone Apposition. This is called the “TENSION SIDE”.

Tension-blasT; Compression-Clast

On the side of the tooth is being moved, you will find OsteoClastic activity that result in bone Resorption. This is
called pressure or Compression side.

March 22/23

Q1.Varices: Lingual Varices under tongue.

Esophageal varices are extremely dilated sub-mucosal veins in the lower third of the esophagus. They
are most often a consequence of portal hypertension, commonly due to cirrhosis; people with esophageal
varices have a strong tendency to develop severe bleeding which left untreated can be fatal. Esophageal
varices are typically diagnosed through an esophagogastroduodenoscopy.

Q2.Hyoid bone

Q3.Why heparin has to be given IV?


A. First bypass
B. Degradation in systemic
C. Acid stability
DR. STRANGE

Q4.Clinical attachment loss: Attachment loss from CEJ to base of pocket

Q5.Shy and submissive child why?


A. Abuse
B. Over protective
C. Autistic

Q6.Reason of wheezing(weird options. Do not remember exactly)


A. Opening and closing of air sac
B. Air out on expiration and come in in air spaces.? O&

narrowing of airways may be missed option

Q7. Doctor need to listen to pulmonary valves sounds. Where should the stethoscope put on the chest
A. Middle sternum on the second intercostal distance
B.Mid clavicular space on the 3 or 4th intercostal distance

Q8. Geographic tongue. Shown on the lateral border of tongue.

Q9. Which ester is metabolized by plasma esterase


Articaine
Mepivacaine
Lidocaine
Bupivacaine
Which amide is metabolized by serum esterases

Q10. Same q painful pain remove quickly once stimulus is removed

A. Reversible
B. Normal
C. Irreversible

Q11.White spot on the single tooth: Trauma, demineralization?

Q12.Fracture of bilateral ramus. Fixation is hard


A. Pull of muscle
B. Damage to never causes paraesthesia

Q13.Pt believes dentist and dental hygienist will harm the pt. infact, they are not

A. Illusion
DR. STRANGE

B. Dementia
C. Paranoia

Q 14. Fibrous dysplasia. ground glass uni or

bilateral treated by bone shaving

Q15. Position of retentive clasp. Give a case.

16. How to fix Protrusive inclination causing


separation of posterior teeth.
ANSWER: altering compensation curve

Q17. Case. Girl 16 girl,missing lat. Incisor. Willing


to get a implants. A lot of missing teeth are seen on x ray. Question about her hygiene. Treatment of missing teeth.

Q18. 6 L N2O. 2.5 L N and 3.5 O2.


How much % is Nitrous
36%
6%
ANSWER: 2.5/6 * 100= 41%
DR. STRANGE

Q19. Wavelength question


Chroma
Saturation
Value
Hue

Q20. How to decrease value. They gave some options like


A. staining
B. add color from wheel

March 24, 2019

1. Question about Gardner Syndrome. What would expect to have?


Answer: Multiple intestinal polyps

2.White spot just one area of root- Turner's hypoplasia


3. Root caries caused by:
A. S. mutans
B. actinomyces

4. First to initiate caries-


A. S. mutans
B. lactobacillus

5. Increase distance two orbits- hyperbolic

6. calculate the loss of attachment. There was gingival recession- So add with pocket depth

7. Eosinophilic granuloma in what?


A. multiple myeloma
B. Langerhans histiocytosis

8. Patient works in nuclear factory. How much of radiation dose /yr is permissible to the dental assistant than
patients?
a. 1/10 x
b. 10 x
c. 2x
d. 1/4x

9. Lots of questions about ethics code

Dentist do publicity violation falls under which code?VERACITY

Dentist force the Patients for specific treatment. Which code of ethics? VERACITY

10. Panoramic identify structure: HYOID BONE

11. Panoramic: AOT

11. Picture: showing lump near gingiva at the Anterior maxillary region: PYOGENIC GRANULOMA
A.SCC
B. papilloma
DR. STRANGE

12. Picture: Tongue


It showed just a slit like color change
Answer: SCC

March 25, 2019


Linis

1.Pregnant women N2O contraindicated when?


A. First trimester only
B. Second trimester only
C. Third trimester only
D. All of them

2.Pregnant 3 trimester doing a procedure, blood pressure decrease, heart rate increase. This is due to? I only
remember orthostatic hypotension

Normal or dehydration?

3. 2 cases about hired an assistant without experience, accidentally she deleted the medical Hx and due to that
patient needs to come back, hygienist call the patient and pt gets angry.. What the dentist should do?

I chose call the patient , explain the reason and ask him to come at his convenient time

4.Same case about dental assisting but this time about perio chart ...

5.aneurysmal cyst :
bone cyst : pseudo cyst

6.in the past 20 years what type of caries has increased? Not sure
A. “root decay”
B. options proximal
C. pits and fissure

7.least rate of recurrence? AOT


8.traumatic neuroma :
edentulous ridge. Denture trauma. Surgical trauma. Mental nerve

9.frey syndrome: crocodile tears

10.feature of fetal alcohol syndrome? Ans: mid face disturbance

11.x-ray of odontoma

11.x-ray of ameloblastoma
DR. STRANGE

12.x-ray of cementoblastoma

13.x-ray of nutrient canal

14.x-day about lateral x-ray maxilla was growing, denture didn’t fit anymore, denture was too tight
Options.
A.Paget's
B.osteitis deformans
C.myeloma multiple (there was nothing in the skull)
D. I don’t remember the last option

15.a lot questions about ept, thermal test,diagnostic


16.which is not true about pin into amalgam? Pin weak amalgam

17.a lot about stains in crown and veneer:


silver whole
Cooper margin

18.chroma=saturation

19. dabigatran ( blood thinner)

20.pt only with brownish maculas skin and oral cavity?


A. Peutz
B Mc Albright
C. neurofibromatosis

21.pt with perio disease, Candida albicans, dry and burn mouth? Chose diabetes mellitus

22. mal seizure : phenytoin


23. no consent: battery
24.dermatologist sent pt to dentist due to suspect ? Chose Gorlin syndrome (due to skin problems)
DR. STRANGE

25. age of HPV;


A. 11-12
B. 9-10
C. 16-18
👍♀️ I knew it was before 18 between 11-12 y.o.

26.non working interference: BULL

27.% of second canal in 2pm MD: 14%

28.Failure of closing in right Cleft lip: failure of joining of lateral nasal and maxillary process??

29.Apert syndrome : acrocephalosyndactyly, malformation of skull, hand and feet. A branchial arch syndrome,
affecting 1st branchial arch the precursor of Maxilla and Mandible.

30.pocket depth

31.biologic width

32.attached gingiva: from groove of free gingiva to mucogingival junction ( they used another word instead of
MGJ)I chose this one.
Other option was from free gingival to MGJ) attached gingiva stars from groove of free gingival

33.If a local anesthetic without a vasoconstrictor is required, then which of the following local anesthetics should be
used? Mepivacaine

33.A lot questions about fluoride

Which one u recommend for a white spot on premolar, no cavitation.


A. No treatment
B.apply varnish fluoride

34.hairy leukoplakia = filiform

35.tooth with isolated deep pocket = vertical root fracture


36.ectodermal dysplasia- abnormal nails, abnormal or missing teeth fewer than normal number of teeth. Cleft
clip. Large forehead.

37.antagonist of hydrocodone = narcotic/naloxone


38.lesion, slow grow about two months on tongue 3mm what to do = excision biopsy

39.lesion persist for more than 2 weeks = must take biopsy

40.cytology show dysplasia what to do next = I chose do biopsy , more option were perform the excisions surgery,
do cytology again to check the diagnostic

I got questions from Rita file, Andres file, and the last remember questions on 23 march
God Bless us

3/25/2019
BASEMA

1- Failsafe NO2 if greater than


DR. STRANGE

A. 20
B. 50
C. 70
D. 90

2-Chemical bond with tooth?


A. GIC
B. Zinc polycarboxylate

3- Gorlin sy: nevoid basal cell carcinoma, multiple tumors, palmar plantar keratosis

4– methotrexat all true except: hair growth

5- Hsv in us: 80-85%

6- all NSAID except acetaminophen

7-four picture of 400 qs: Scleroderma, ear lop, geographic tongue ....

8- dentures anterior teeth touch each other strongly: decrease horizontal overlap( only makes sense)

9- all give support to u lip in denture: Arch shape- flange thickness- shape of the anterior teeth-occlusal plane

10- clonidine what receptors(mao): Alpha 2

11- a lot qs NSAID ( cox)

12- what not anti inflammatory? Acetaminophen

13- Sulfonamide: PABA

14- pt has cheilitis , fever, and has no allergy what anti inflammatory? Penicillin v

15- kids what deicide amount of LA? Weight

17- clido dysp, ectodermal ( oligo & anodontia 2qs)

18- pt management

19- least cause make u extract 2 primary molar in case no 2nd premolar:
A. ankylosis
B. resorption root
C. big caries

20- alot qs about ankylosis indirect way that presence in x ray radiolucent or opaque

21-cellulitis patient case what u will do 1st

A. Give antibiotics only


B. Give antibiotics and incision & drainage
C. Incision and drainage and rct cause there is infection
D. Refer him to oms

22- poster ortho extrusion cause relative intrusion for ant teeth or absolute (. An 1)

23- what primary tooth if lost it will be crowding? Canine


DR. STRANGE

24- buccal eruption for L canine leads to:


A. deep bite
B. bone recession

25- if lesion not go after how many make biopsy: 10-15 days

26- implant to implant distance 3 mm

27- least distance in buccal lingual for implant diameter 4 mm:


A. 5
B. 7
C. 10 no 6

28- calculate the loss of attachment. There was gingival recession- SO add with pocket depth and: 4
29- a lot reversible irreversible pulpitis percussion periapical periodontitis ,all easy
29-Cleidocranial
30- cleft lip 6-9 months in uterus
31- bleach - wait 1 week then veneer 2-3 times
32- If a particular test is to correctly identify 95 out of 100 existing disease cases, then that test would have a

A- Specificity of 95%
B- Sensitivity of 95%
C- Positive predictive value of 95%
D- Validity of 95%

33-Disadvantages of oral sedation include which of the following?

A- Multiple drugs needed for proper sedation


B- High incidence and severity of adverse reactions
C- Unpredictable absorption of drugs from the GI tract
D- Short duration of action

34-Most resistance from features in fixed dental prostheses such as boxes and grooves should ideally be located?

A- Facially
B- Lingually
C- Occlusally
D- Proximally

35- Which permanent teeth are most often congenitally missing?

A- Lateral incisors
B- Canines
C- upper Premolars
D- lower molar

36- how do u know that pt will follow on instructions: pt said I will put floss with my evening medication

37- water fluoride not make any harm to teeth:

A. 0.5-0.1
B. 0.6-1.2

38- chroma saturation, hue wavelength

39- 14 yo pano fibrous dysplasia on her right side

40- Epileptic status - midazolam


DR. STRANGE

41- kaposi sarcoma caused by HIV? No Hpv: HSV8


42- naloxone MOA 2Qs: Block or reverse the effect of Opioid drug

43- x- ray from 8 to 16 how much density?


A. ¼
B. 4
C. 2
44- x-ray collimation: reduce the spot size
45- X-ray : scattered from pt
46- what MRI consist of: radio waves
47- alot pathology described without photos
48- most tooth crown root fracture: maxillary incisors
48- craze lane : transillumination
49- one q about cholesterol lowering in blood medicine.: Statin group?

For day2 Andres file very important

1-The rubber dam does NOT protect patients from which of the following?

-Unnecessary chemical exposure


-Aspirating objects such as an inlay
-Oral lacerations from dental instruments
-Mercury vapors when removing old amalgams

2- what not triggering sickle cell anemia

- cold

- trauma

- NO2

3- if we extend posterior buccal border wt muscle consider? Masseter

4- pt with erythema on tongue multiple places surrounded by white border change it place?
Erythema migrans( same words)

5-for last q treatment all except?

Chlorhexidine, nystatin, keep good oral hygiene....

6- systemic and local antifungal: Ketoconazole BOTH Topical for Mucocutaneous or Oral for systemic
7- antifungal systemic in case: Ketoconazole, fluconazole, Itraconazole, Voriconazole, posaconazole

8-impression material ...? Thermoelasticity

10- all this contains opaque except:

AOT
odontoma
DR. STRANGE

Dentigerous cyst
Calcified odontogenic cyst

11- q in case 21 yo female has white spot on central incisors incisive edge( decalcification like) we can use all except
:
- Fluoride varnish
- Systemic fluoride
- Florid something but not systemic

13-The prostaglandin analog misoprostol (Cytotec) is most commonly used in treating gastric ulcers associated with

A- Chronic use of anti-inflammatory drug


B- Chronic use of tobacco products
C- Hypersecretory conditions such as Zollinger-Ellison syndrome
D- Hyperactivity of the gastrin-secreting chief cells

14-Alcoholic patient…which test before surgery?


15- how much middle third reduction for veneer?
16- pt has a lot root caries which material filling u use: composite- glass ionomer , amalgam

17- pt has limited attached gingiva near 2nd m molar which has recession in distal what flap not to do? MWF- laterally
displaced flap- gingivectomy- distal wedge

MARY’S RQS

1. Two different cases with a 17-year old boy with a broken tooth and lip laceration, same question: how to adjust
time exposure for an xray to find the tooth fragment in the lip?: Decrease time by 25%

2. What is important in single implant tooth replacement:


A. smooth interface
B. connecting the implant to neighboring teeth
C. broad contact with neighboring teeth?

3. Chronic periodontitis?
A. white male
B. AA male
C. asian female
D. native american male
E. Black male(TWICE)

4. Antibiotic in gingival crevicular fluid that fights perio bacteria? Doxycycline

5. After extraction, how you can control bleeding? (normal conditions) with simple gauze pressure

6 Water fluoridation in a community, which two ethical principles are in conflict? autonomy and beneficence

7. Green and orange discoloration of anterior teeth due to? bad oral hygiene

8. Cord with epinephrine, what can cause?


A. Tachycardia
B. local necrosis
DR. STRANGE

9. Hearing impaired patient, what to do to establish good relationship with the patient?

A. speak directly to the interpreter


B. speak to the patient and allow time for interpretation (went with that)
C. cancel all background noise
D. rely on patient lip-reading abilities

10.Macrolide antibiotics are usually avoided in asthmatics because:

A. it can precipitate asthmatic attack


B. it interacts with theophylline
C. such patients are usually allergic to penicillins
D. such patients show penicillin resistant infections

11. Osteotomy Indications (questions about anterior open bite)

12. Mandibular third molar impacted extraction, distal root got missing, where it is most likely:

A. retropharyngeal,
B. submandibular
C. Submandibular

13. A child comes to your office with a neighbour, he states that he was playing in the yard and injured himself, they
gave me the class of injury, i think it was Ellis III but it wasn’t relevant for the qs. The adult states they could not
contact his mother, the kid says he has a dentist but doesn't remember his name. What do you do?

A. do the treatment
B. look for the dentist and try contact him,
C. try contacting the parents (picked this, in the case they said they tried to contact mom)
D. send to emergency room

14. Picture of a kids’ primary front teeth, literally eaten by caries, with two sticking out fistulas - asked what are those
changes on mucogingival junction (pus draining fistulas): Parulis

15. Big alveolar defect, which graft has the best prognosis? went with autologous bone

16. Peutz-Jeghers and Gardner –Intestinal polyps

17. Completely intruded primary tooth that does not impinge on permanent Which is the correct treatment?

A. ortho tx
B. surgical
C. Observe

18. Kids dexterity to brushing teeth on his/her own? 2-5 they gain skills;6-9 they can brush on their own; 8 they
can floss.

19.Latex gloves, which impression material can mess up?

A. Pvs
B. addition silicon

20. Arcon and non-arcon, difference: picked something with more movement abilities

21. Adolescent son comes to the office with a father that doesn’t speak English, what is best to do?

A. use a certified call-center line


B. relay on son translation
C. translate on the internet
DR. STRANGE

22. Mucous extravasation phenomenon on lower lip due to:

A. sialolith of small gland


B. trauma

23. Biological width, how much is it and what does it consist of ( Concept)
JE 0.97mm +CT 1.07 mm=2.04 mm

24. which is not an adverse effect of the cocaine?

A. miosis
B. mydriasis

25. H-antagonist work by:

A. blocking histamine at the receptor


B. preventing histamine synthesis
C. physiological antagonism

26. You want to do a study to compare the mean DMFT of girls and boys at school, which test:

A. chi square
B. t-test

27. Which is the most common in primary teeth:

A. flush terminal
B. mesial step
D. distal step
E. class I,
F. class II

28. what means Bimaxillary effect? BOTH ARCH ARE PROTRUDED something with relation of both jaws to the
cranial base

29. Case, patient with porcelain crown, a lot of caries, what would you suggest?

A. all overnight treatments with fluoride rinse (would be best)


B. acidulated
C. varnish fluoride every day
D. 0.4% stannous

30.Picture of kids teeth and gingiva, very swollen, in the history it says he’s been treated for skin infections and
gingivitis look like that for a couple of months now, it bleeds, what is most likely diagnosis?

A. myeloblastic leukemia,
B. pyogenic granuloma

31. What we don’t asses before implant treatment planning:?

A. Age
B. smoking history

32. Best prognosis for implant:

A. anterior mandible
B. posterior mandible
C. anterior maxilla
DR. STRANGE

D. posterior maxilla
E. Mand ant, d2 type bone

33. Pregnant, first trimester, picture of pyogenic granuloma, what to do:

A. wait till second trimester


B. Biopsy
C. root planning

34. Piezoelectric ultrasonic device, what movements:

A. Elliptic
B. Ovoid
C. Linear
D. circular

35. Nitrous oxide, when contraindicated:

A. first trimester only


B. second trimester only,
C. third trimester only,
D. all trimesters

36. Contraindicated with asthma, what can precipitate the attack: NSAIDs

37. Pregnant, third trimester (36 weeks was given), why we don’t administer ibuprofen:

A. premature ductus arteriosus closure


B. cleft lip and palate
C. spina bifida
D. tachycardia in the mother

38 MOA of ibuprofen:
A. reversible platelet inhibition
B. irreversible platelet inhibition
C. extrinsic pathway
D. intrinsic pathway

39. Kid after baseball trauma, tooth is broken (dentin with exposure), no signs of laceration, tooth fragment was not
found, what best for diagnosis:

A. Panoramic
B. periapical with different angulation

40. MRI is best for TMJ to see well:

A. Disc
B. condyle

41.Most common among population:

A. cleft lip and palate


B. DI
C. AI
D. DD

42.Seen in osteoporosis: thin trabeculae

43. separate canals in: fusion


DR. STRANGE

44.Down syndrome features except?

A. delayed eruption
B. perio issues
C. high caries rate
D. macroglossia

45.Temporal arteritis what is the most likely outcome?

A. retro orbital pain


B. vision loss
C. temporal pain

46.Border molding on buccal posterior, near masseter, what movements:

A. lateral tongue movements to both cheeks


B. closing mandible against pressure
C. lateral jaw movements

47.what is false about Combination syndrome? Increase vertical dimension

48. Hydrolysis CO2 CO2 + H20 <=> H2CO3(carbonic acid) what is catalyst? Carbonic Anhydrase

49. Most common association of osteogenesis imperfecta with

A. Amelogenesis
B. Dentinogenesis
C. Dentinal dysplasia

50. Cause of isthmus fracture in amalgam? insufficient depth

51.Which relationship of primary molars can lead to Class 2 malocclusion? distal step

52. amalgam, you notice a small chip on margin - 0,5 mm What you'll do?

A. To replace a part of amalgam


B. To replace all amalgam
C. Leave it and observe

53. During endo you meet a ledge inside the canal What to do?

A. Take a larger file


B. Leave and obturate before the ledge
C. Try to remove ledge
D. Take a smaller file and try to bypass

54.During an IAN not been able to achieve proper anesthesia means it went to which accessory innervation: Long
Buccal

A. Mylohyoid
B. Auriculotemporal
C. Two more nerve

55. Most radioresistant

A. Nerve
B. Striated muscle
DR. STRANGE

56. Cephalometric x-ray, and in an upper right corner there is an image of a ruler, it's just a vertical part of ruler line.
What for the ruler here?

A. For cephalometric measuring


B. For determining of magnification of the image
C. For determining relation of upper part to lower (face)

57. The amount of radiation on a panoramic RX is compared to?

A. A CT scan
B. MRI
C. 4 bitewings
D. Full mouth series of rx

58. most common emergency- syncope

59. LA ques..3 yrs 16 kg, max dose: option was 72 (no 70.4 in option)
60. LAP: t/t: Srp + Ab
61. def given: Attrition
62. LAP caused by: AA

63. perforation cavity on max ant: buccal, mesial in RCT?

64..unbundling ques: dentist and the x ray things.

65. antibiotic for sinusitis: Amox + clavulanate potassium

66. Safe drug for renal: acetaminophen

67.Drugs for angina: propranolol (other nitrates, Ca channel wasn’t in options)

68. Drugs not given in angina: Thiazide

69.Plaque index used for: Pt motivation

70.Nitrous oxide adv: to decrease gag reflex

71. Distal step leads to: Class 2

72. Oxycodone + Acetaminophen which class: Sch 2 Percocet

73. Sch 2 drug refill: must be : Prescription

74.Nitrous oxide C/I: nasal congestion

75. Supporting area sup in rpd

76. What’s not included in CAMBRA Caries assessment?

A- Genetic factors
B- Biological factors
C- Disease factors
D- Risk factors

77. trepanation

A. Open the floor of the pulp cavity


B. Open the pulp cavity and apical constriction
DR. STRANGE

C. Mucosal and periosteal incision at the apex


D. Working thru the cortical bone to elevate pressure at the apex

78.how do you determine the path of insertion on an abutment

A. One eye and a mirror


B. Two eyes to see the undercuts
C. Study models

79.hypodontia- initiation stage

80.port wine stain -sturge weber syndrome

81.craniofacial deformities except: tetralogy of fallot

82.major connector:
A.rigidity and support
B. Rigidity and retention

83.you realize the gutta percha is above the apex what is the first that you do?

A. Tell the situation to the patient B. periodical observation

84.anterior infraorbital notch of facebow question

A. Distance between infraorbital rim and occlusal plane


B. Relation of occlusal plane to infraorbital rim hinge axis

85.cytological smear comes back positive with some cell what do you do next?

A. Biopsy
B. Second cytology to confirm the diagnosis
C. Excisional
D. Incisional

86.arch perimeter is measured on the mesial of:

A. 1 pm
B. 2 pm
C. 1 m
D. 2m

87.indirect retainer what does it do? The answer was a little long but it was along the lines of preventing dislodgement
of distal extension

88.when you do flap surgery you cut thru healthy pdl.when that heal and connects cementum to bone with sharpey
fiber what is it called?

A. Regeneration
B. Reposition
C. Replacement.
D. Reattachment

89. How is the way for transmission hepatitis A? food

90. Gingivectomy does what? Suprabony pockets and suprabony periodontal abscess and remove gingival
hyperplasia

91. 2 molar of mandibular has less keratinized gingiva deep pocket on the distal what don’t you do?
DR. STRANGE

A. Distal wedge
B. Gingivectomy

92.menarche is related to skeletal growth


Which of the ff. Describes growth after menarche:

A. increases
B. Stays constant
C. Stops
D. Decreases?

93. how you can probe vertical dimension in a total denture: Sibilant/ Linguo Alveolar sound
94.what changes over time : Chroma
95.pseudomembranous colitis: clindamycin
96.tetracycline except: bactericidal

97.horizontal fracture of the root of the tooth what do you do?


A. rct
B. Preserve vitality
C. Splint

98. beclomethasone: short acting

99.1st molar calcification ends? 2-3 yrs

100.pain from muscles of mastication

A. non-steroidal plus muscle relaxants


B. Non-steroidal plus night guard
C. Muscle relaxants plus night guard

101.fetal alcohol syndrome?- midface

102. Anesthesia w/o vasoconstriction -: mepivacaine

103. Opioid antagonist: Naloxone

104. BDZs antagonist: Flumazenil

105. year female with high fever, lymph nodes tender and erythema of oral and pharyngeal mucosa.
a. Strep pharyngitis

106. all are benefits of 3/4 crown as compared to full crown except

A. better retention?
B. Better marginal access. can perform diagnostic testing
C. esthetic.

107.Another was rheumatic disease most common leads to:

A. joint effusion?
B. cardiac murmurs
C. bone infection

108.Best method for diagnosis of NUG?

A. bacteriological examination
B. ulcers at interdental papilla
DR. STRANGE

109) Another question was most common organism in chronic periodontitis

A. facultative gram negative


B. anaerobic gram negative
C. facultative gram aerobic?
D. aerobic gram positive

110) Patient want to change his habit of smoking, how will you address this patient
A. identity patient interests in habit changing

111. how many cutting edges in gracey curette, One lateral cutting edge in gracey

112.What is Y Line in maxilla radiograph) it is comprised of floor of maxillary sinus and nasal fossa

113.What is the disadvantage of systematic desensitization?

A. Cost effective
B. Time consuming

114.epiphyseal plate resembles: synchondrosis

115. acute adrenal crisis ( hypotension)

116. epinephrine interacts with : thyroid

117- Most common age for primary teeth fracture? 2-4

118- Reason for children to have fracture\luxation injuries? Coordination \balance

119.Radiation effects which surface of the teeth more? Cervical

120- Which cyst is not related to bone? Nasolabial cyst

121. Impression material shrinkage in moisture is known as? Syneresis

121. difference between angina and MA: thrombosis

122. panoramic, elongation of styloid process, asking for what syndrome? eagle syndrome was patient in pain
when open his mouth

122. mandibular nerve exit the skull through what foramen: Ovale

123- xylitol for caries prevention. How to use : (varnish, mouth rinse, gum)

124. why the anterior face infections are dangerous because they will spread infection to what structure? cavernous
sinus

125.the most harmful force to single implant (vertical, horizontal, oblique, axial)

126- which will least resist force best single implant 4mm, 5mm, two splinted? 4mm splinted

127- brown discoloration of pfm? Microfracture was the best choice to me: microleakage

128- marginal discoloration at pmf (cooper, silver, nickel)

129- why chroma cobalt alloy will resist tarnish and corrosion (surface oxide layer, noble)?

130- sensitivity to cold and pressure two weeks after pfm? Vertical fracture or occlusal trauma
DR. STRANGE

131.epinephrine and levonordefrin act on which receptor to cause hypertension: alpha 1 receptor

132.What is penumbra? Penumbra is lack of sharpness of the film

133.Deepest part of rest

A. Occlusal: 2mm
B. Marginal ridge : 1.5 mm

134. Distance between matrix band and marginal ridge: 1mm greater marginal ridge

135.Patient has lithium in drug history. Which disorder does he: have Bipolar

136.First pass metabolism Liver

137.How does intrapulpal anesthesia work Back pressure

138. 2 proximal cavities Class 3 (Prepare larger, restore smaller)

139. Difference between hema and tegma ANS viscosity

Tegma- Strength, and color stability. Hema is monomer for viscosity

140. Nistatin in HIV patient : HIV patient with oropharyngeal candidiasis

141. What happens after occlusal trauma with periodontal ligament? Widening of PDL

142. Which is not a reason to extract 3rd molar?

A. Prevent crowding in anteriors


B. Recurrent operculitis

143. You do gold resto for patient, immediately after it pt feels a pain reason? Galvanic shock

144. Treatment of alveolar osteitis??: put a palliative dressing

145. Informed consent is type of Autonomy

146. most common cells in gcf Neutrophils

147. . what would least improve retention of a crown prep. finish line?

148. What test for what disease should the doctor perform every year: Tuberculosis

149. Caries on a patient extends subgingivally what would be the approach Crown lengthening

150. Gingivoplasty can be done in = ANUG

151.Horizontal root fracture in middle third and the crown is slightly mobile = Splint and recall patient

152. Which of the following dental conditions is often found in Down's syndrome patients?

A. Mulberry molars
B. Supernumeraries
C. Short, conical roots
D. Hutchinson's incisors

153. Pemphigus: suprabasilar vesicle and acanthosis


DR. STRANGE

155.Treatment without consent=Battery

156.Patient had a root canal 2 years ago and now you see an apical radiolucency. What to do pt asymptomatic
=Retreatment if the tooth becomes symptomatic

157. Dose of hydrocortisone taken per year that will indicate have adrenal insufficiency and need supplement dose
for surgery

- 20 mg 2 weeks for 2 years

158.Damage to biological systems from ionizing radiation is due primarily to which of the following?
A- Transmutation of a key atom
B- A direct hit on a key atom or molecule
C- Radiolysis of water molecules

159.Root caries = Soft tissue in dentin

160 Most common reason for Dilantin treatment?

A- Grand mal
B- Petit mal

161. Pigmentation of the face, lips and oral cavity and intestinal polyps = Peutz Jeghers syndrome

162.All will be included in the ortho informed consent except

A. Ortho treatment can bend roots of the teeth?


B. Caries and gums diseases can happen during ortho treatment
C. Injury to the nerve due to any previous accident can be increased
D. During ortho Pt will have to wear mouthguard during sports

163.Muscles to elevate mandible except? Pterygoid lateral

164.Chroma is the degree of color saturation.

165. In a primary tooth, apical infection on the radiograph is usually where? In the furcation

166- Critical pH for enamel demineralization = 5.5

167.Under which principles of informed consent of treatment, a dentist is NOT required to inform a patient about
which of the following?

a) Alternative treatments available by a specialist


b)The risk associated with non-treatment
c) Potential risks of treatment
D) insurance co-payment for treatment

FORESEEABLE RISKS & OCCUR

168.Which fluoride for porcelain something = Neutral Sodium fluoride

169.Best prognosis to need a bone graft = 3-walls

170.Blue sclera seen in : osteogenesis imperfecta

171. Primary bacteria for the initiation of caries? A- Strep. Mutans

172. Which papilla is affected in geographic tongue? A- Filiform papillae


DR. STRANGE

173. Which teeth most likely to be crowded/blocked out of mandibular arch?

A- Incisors
B- 1st premolars
C- 2nd premolars
D- 1st molars

174. What doesn’t affect implant treatment planning?

A- Age
B- Smoking history
C- Quality of the bone
D- Location

175. Which teeth most likely to be crowded/ blocked out of maxillary arch? A- Canines

176. Teeth least likely to have two canals? A- Maxillary incisors

177. Most common reason of maxillary tori removal?: Palatal are usually NOT removed mandibular are usually
removed. Possible answer is Extends so far posteriorly interfering with the posterior palatal seal.

A- Prosthetic treatment
B- Malignancy transformation
C- Hyperplastic papillary tissue formation

178.Patient is opening her mouth, click when 25 mm opened, then second click when it’s 5 mm until full closure -
what is the reason for second click?
A- Displacement of the disc from anterior position to being back on the condyle
B- Displacement of the disc posterior to the condyle

179.Laser in periodontal surgery, recent studies shows that?

A- It makes the area sterile


B- helps healing faster
C- Prevent bleeding
D- New Attachment
E- Other options

180.Class II furcation, what you wouldn’t put in a treatment plan?

A- GTR
B- extraction with implant placement
C- hemisection with prosthetic crown placement
D- converting a class II to a class I furcation

181. First-pass effect?

A- liver metabolism
B- intestinal absorption
C- kidney clearance
D- Other options

183.. What has not been associated with periodontal disease?

A- Nutritional deficiencies
B- Smoking
C- Age
D- Genetics
184.. Which Antifungal is available in troches? A- clotrimazole
185- Occlusal reduction of porcelain crown?
DR. STRANGE

A- Flat and rounded


B- Sharp and angled
C- Same as the occlusal anatomy

186. Picture of the tongue, lateral side with bluish lesion, patient states it is present for a 5 years now, growing but
really slow?

A- Hemangioma
B- Veracities
C- Hamartoma
D- Kaposi sarcoma

187. Most common seizure in kids:

A- Febrile
B- Absence
C- Grand mal
D- Epileptic

188. Most dental financing is from?

A- Self-pay
B- Medicare
C- Insurances

189. Firm painful nodule next to mental foramen?

A- Traumatic neuroma
B- Neurofibroma
C- Other neuromas

190. Hairy tongue, which papillae affected?

A- filiform
B- fungiform
C- foliate

191.You are prepping the tooth for a crown, but realized, that the step is only 1 mm away from the alveolar crest,
what do you do?

A- crown lengthening
B- nothing
C- Other options

ANS A - Biological width is compromised

192. Plaque index is used mainly for:

A. Patient motivation
B. Gingivitis estimation
C. Calculus index
D. Other options

193. A child’s behavior problem can be managed by desensitization if the basis of the problem is

A- Pain
B- Fear
DR. STRANGE

C- Emotional
D- The parents

194. Disadvantages of oral sedation include which of the following?

A- Multiple drugs needed for proper sedation


B- High incidence and severity of adverse reactions
C- Unpredictable absorption of drugs from the GI tract
D- Short duration of action

195. Auxiliary resistance from features in fixed dental prostheses such as boxes and grooves should ideally be
located?

A- Facially
B- Lingually
C- Occlusally
D- Proximally

196. Which of the following would be LEAST likely to lead to the development of root surface caries on facial
surfaces?

A- Low salivary flow


B- Elevated levels of sucrose consumption
C- Streptococcus sanguis dominating adjacent plaque
D- History of head/neck radiation therapy

197. The patient should sign the informed consent for surgery

A- At the time of filling out history and insurance forms


B- After medical history and physical examination
C- After formulation by dentist of a surgical treatment plan
D- After a full discussion of the surgical treatment plan

198- Which of the following does NOT produce a pharmacologic decrease in saliva production?

A- Atropine (AtroPen)
B- Scopolamine (Transderm-Scop)
C- Pilocarpine (Salagen)
D- Glycopyrrolate (Robinul)

199. The rubber dam does NOT protect patients from which of the following?

A- Exhaled nitrous oxide


B- Unnecessary chemical exposure
C- Aspirating objects such as an inlay
D- Oral lacerations from dental instruments
E- Mercury vapors when removing old amalgams

200. Which of the following contributes most to a successful pulp capping procedure?

A- An isolated field
B- Absence of bleeding
C- An exposure of less than 0.75 mm
D- Use of calcium hydroxide

201. Which of the following has the greatest impact on the success of a periodontal flap procedure?
DR. STRANGE

A- Type of initial incision


B- Position of the flap at closure
C- Level of postoperative plaque control
D- Postoperative width of attached gingival

202. The most common form of periodontal disease seen in school-aged children is

A- Aggressive periodontitis
B- Marginal gingivitis
C- Primary herpetic gingivostomatitis
D- Gingival hyperplasia

203. Who is associated with the least risk of inhaling N2O?

A- Dentist
B- Patient
C- Dental hygienist
D- Dental assistant

204. Case. Patient comes with bilateral bone expansion, and complains about Dentures won’t fit? A- Paget’s
disease

205. Open bite more than 8 mm, what you'll do?

A- Le Fort 1 osteotomy
B- Sagittal osteotomy
C- Vertical osteotomy
D- Orthodontic

206. What do you check in the wax try in Except:

A. Aesthetics
B. Vertical dimension
C- Occlusion
D- Obtain facebow record

207. What cannot be on tongue?

A- Ectopic thyroid
B- Peripheral giant cell?
C- Pyogenic Granuloma

208. Less recurrence tumor:

A- Odontogenic keratocyst
B- Compound odontoma
C- Ameloblastoma

209. what you find in area of tension:

A- Increased fibroblast activity


B- increased number of osteoclast ?
C- Other options

210. Papilloma? A- Warty like Lesion

211. All of the following are associated with Ludwig’s Angina, EXCEPT?
DR. STRANGE

A- Retropharyngeal Space
B- Submandibular
C- Sublingual
D- Other options

212.Difference between primary and secondary occlusal trauma?A - PDL involvement and remaining amount of
supporting bone

213. All of the following are characteristics of Sodium hypochlorite EXCEPT?

A - Chelating agent
B- Irrigation
C- Lubricant
D- Disinfectant

214. Natural wear down of teeth surfaces? A- Attrition

215. Size and shape?

A- Morphodifferentiation
B- Histodifferentiation
C-Apposition

216. How to increase retention/resistance on Short teeth? A- Proximal grooves

217.Colored tiny spot on the internal surface of lower lip, what’s the most probable diagnosis?

A- melanotic macule
B- melanotic nevus
C- Other options

218- What’s the cause of the folds on the tongue? ( it was unilateral )

A- teeth indentation
B-Trauma
C- Other options

Day 2:

I took the same cases of Jessica gardner 1,2,3,4, and 7

S-ar putea să vă placă și